Nursing Final

Ace your homework & exams now with Quizwiz!

A nurse is preparing to administer albuterol 4 mg PO every 6 hr. Available is albuterol syrup 2 mg/5 mL. How many mL should the nurse administer per dose?

10 mL

a nurse is preparing to administer haloperidol 3 mg im for agitation. available is haloperidol 5 mg/ml for injection. How many mL should the nurse administer?

0.6 mL

a nurse is preparing to administer heparin 15,000 units subcutaneous every 12 hr. available is hepari injection 20,000 units/ml. how many ml should the nurse administer per dose?

0.75 mL

1 mg

1,000 mcg

A nurse is preparing to administer tobramycin 2.5 mg/kg IM to a preschooler who weighs 20 kg. Available is tobramycin injection 40 mg/ml. How many ml should the nurse administer per dose?

1.3

a nurse is preparing to administer meperidine 100 mg IM stat. Available is meperidine injection 75 mg/ml. How many ml should the nurse administer?

1.3 mL

a nurse is preparing to administer chlorpromazine 40 mg im to a client. available is chlorpromazine 25 mg/ml. How many mL should the nurse use?

1.6 mL

A nurse is preparing the administer cephalexin 500 mg PO every 6 hr. Available is cephalexin suspension 250mg/5mL. How many mL should the nurse administer per dose?

10 mL

A nurse is converting an infant's weight from lb to kg. If the infant weighs 6 lb 2 oz, what is the infant's weights in kg?

2.8 kg

A nurse is preparing to administer amozicillin 30 mg/kg/day PO divided in equal doses every 12 hr to an infant who weighs 5.5 kg. Available is amixicillin suspension 125 mg/5 ml. How many ml should the nurse administer per dose?

3.3 mL

A nurse is preparing to administer valproic acid 45 mg/kg/day PO divided in equal doses every 8 hr to a child who weighs 66 lbs. Avaliable is valproic acid syrup 250 mg/5 ml. How many ml should the nurse administer per dose?

9 mL

A nurse is preparing to administer ceftazidime 40 mg/kg IV bolus every 8 hr to a toddler who weighs 22 lb. available is injection of 40mg/mL how many mL should the nurse administer per dose?

10 mL

Amoxicilin 250 mg/125/5ml (the picture one)

10 mL

A nurse is preparing to administer cefdinir to a child who weighs 44 lb. the medication reference recommends administering 7 mg/kg PO every 12 hr. How many mg should the nurse plan to administer per dose?

140 mg

1 kg

2.2 lbs

A nurse is preparing a medication and calculates the dosage as 0.893 mL. Rounding this amount to the nearest hundredth, the nurse should administer how many mL?

0.89 mL

A nurse is preparing to administer regular insulin 15 units and isophane insulin 40 units subcutaneous at 6:30 a.m. At which of the following times should the nurse administer the medication? 0430 1630 0630 1430

0630

8 oz

1 cup

A nurse is preparing to administer diphenhydramine 25 mg PO to a client. The amount available is diphenhydramine elixir 12.5 mg/5 mL. How many mL should the nurse administer?

10 mL

Acetaminophen (Tylenol)

Drug class: Nonopioid Analgesics MOA: activation of descending serotonergic inhibitory pathways in the CNS uses: pain and fever

30 mL

1 oz

15 mL

1 tbsp

A nurse is preparing penicillin G benzathine 1,200,000 units IM. Available is penicillin G benzathine injection 600,000 units/ml. How many ML should the nurse administer per dose?

2 mL

a nurse is preparing to administer methlyprednisolone acetate 40 mg im daily. available is methylprednisolone acetate injection 20 mg/ml. How many mL should the nurse give?

2 mL

A nurse is preparing to administer hydromorphone 4 mg PO every 4 hr PRN for pain. Available is hydromorphone oral liquid 1 mg/1 mL. How many mL should the nurse administer per dose?

4 mL

1 L

1000 mL

A nurse is preparing to administer famotidine 0.5 mg/kg PO to a child who weighs 50 lb. How many mg should the nurse administer per dose?

11.4

A nurse is preparing to administer erythromycin ethylsuccinate (EryPed) 50 mg/kg/day PO divided in equal doses every 6 hr to a toddler who weighs 32 lb. Available is erythromycin ethylsuccinate suspension 200 mg/5 mL. How many mL should the nurse administer per dose?

4.5 mL

A nurse is converting an infant's weight from lb to kg. If the infant weighs 10 lb 4 oz, what is the weight in kg?

4.7

A nurse is preparing to administer midazolam IM to a client who is preoperative and weighs 132 lb. the medication reference states the safe dosage range is 0.07 to 0.08 mg/kg. that is the max safe dose the nurse should expect to a administer

4.8

A nurse is preparing to administer epinephrine hydrochloride 0.5 mg subcutaneous stat. How many mcg should the nurse administer?

500 mcg

A nurse is preparing to administer cephalexin 0.25 g PO every 6 hr. Available is cephalexin 500 mg tablets. How many tablets should the nurse administer per dose?

0.5 tablets

A nurse is preparing to administer doxycycline 100 mg PO every 12 hr. Available is doxycycline 50 mg tablets. How many tablets should the nurse administer per dose?

2 tablets

A nurse is preparing to administer quinapril 40 mg PO daily. Available is quinapril 20 mg tablets. How many tablets should the nurse administer daily?

2 tablets

a nurse is caring for an infant who weighs 11 lb 4 oz. what is the infant's weighs in lb?

11.25 lb

A nurse is preparing to administer methylprednisolone 4 mg PO daily. Available is methylprednisolone 8 mg tablets. How many tablets should the nurse administer daily?

0.5 tablets

A nurse is preparing to administer gentamicin to a client who weighs 165 lb. the medication reference recommends administering 2 mg/kg IM every 8 hr. how many mg should the nurse plan to administer per dose?

150 mg

A nurse is preparing to administer digoxin 250 mcg PO daily. Available is digoxin 0.125 mg tablets. How many tablets should the nurse administer?

2 tablets

A nurse is preparing to administer amoxicillin 500 mg PO every 8 hr. Available is amoxicillin oral suspension 250 mg/5 mL. How many tsp should the nurse administer per dose?

2 tsp

A nurse is preparing to administer diphenhydramine 25 mg PO every 6 hr. Available is diphenhydramine syrup 12.5mg/5mL. How many tsp should the nurse administer per dose?

2 tsp

A nurse is preparing to administer methadone 2.5 mg PO every 8 hr. Available is methadone 5 mg tablets. How many tablets should the nurse administer per dose?

0.5 tablets

A nurse is caring for a client who states that his provider told him he is at risk for anaphylaxis following administration of amoxicillin and that he does not understand what this means. Which of the following is an appropriate response by the nurse? "Anaphylaxis is a predictable and often unavoidable secondary effect that can occur at a usual therapeutic dose." "Anaphylaxis will cause you to experience withdrawal symptoms when you discontinue taking the medication." "Anaphylaxis is an unusual response that can occur due to an inherited predisposition." "Anaphylaxis is a severe hypersensitivity or allergic reaction that is life-threatening."

"Anaphylaxis is a severe hypersensitivity or allergic reaction that is life-threatening."

a nurse is preparing to administer purified protein derivative 5 tuberculin units tu intradermal to a client. Available is 5 tu/0.1mL. How many mL should the nurse administer?

0.1 mL

a nurse is preparing to administer enoxaparin 30 mg subcutaneous to a client. Available is enoxaparin 60 mg/0.6 mL. How many mL should the nurse administer?

0.3 mL

A nurse is preparing to administer epoetin alfa 50 units/kg subcutaneous to a preschooler who weighs 40 lbs. Available is epoetin alfa 2,000 units/ml. How many ml should the nurse administer?

0.45

A nurse is preparing administer lorazepam 2mg iv bolus 20 min prior to surgery. Available is lorazepam injection 4 mg/mL. How many mL should the nurse administer?

0.5 mL

a nurse is preparing to administer calcitonin 100 units subcutaneous to a client. available is calcitonin 200 units/ml. how many mL should the nurse administer?

0.5 mL

a nurse is preparing to administer digoxin 0.125 mg iv bolus stat. available is digoxin injection 0.25 mg/ml. how many ml should the nurse administer?

0.5 mL

a nurse is preparing to administer diphenhydramine 25 mg iv bolus stat. available is diphenhydramine injection 50 mg/ml. How many mL should the nurse administer?

0.5 mL

a nurse is preparing to administer morphine 5 mg subcutaneous to a client. available is morphine 10 mg/ml. how many ml should the nurse administer?

0.5 mL

A nurse is preparing to administer trazodone 25 mg PO at bedtime. Available is trazodone 50 mg tablets. How many tablets should the nurse administer per dose?

0.5 tablets

A nurse is transcribing medication prescriptions for a group of clients. Which of the following is the appropriate way for the nurse to record medications that require the use of a decimal point? .4 mL 0.6 mL 8.0 mL 125.0 mL

0.6 mL

a nurse is preparing to administer diazepam 3 mg iv bolus every 3 hr prn anxiety. Available is diazepam injection 5 mg/ml. How many ml should the nurse administer per dose?

0.6 mL

a nurse is preparing to administer meperidine 75 mg subcutaneous every 4 hr prn pain. Available is meperidine injection 100mg/ml. How many ml should the nurse administer per dose?

0.75 mL

3 tsp

1 tbsp

A nurse is preparing to administer codeine oral solution 30 mg PO to a client. The amount available is codeine 10mg/5mL. How many Tbsp should the nurse administer?

1 tbsp

5 mL

1 tsp

A nurse is preparing a medication and calculates the dosage as 1.42 mL. Rounding this amount to he nearest tenth, the nurse should administer how many mL?

1.4 mL

a nurse is preparing to administer diazepam 7 mg iv bolus every 4 hr prn muscle spasm. Available is diazepam injection 5 mg/ml. How many mL should the nurse administer per dose?

1.4 mL

a nurse is preparing to administer methylprednisolone acetate 120 mg im daily. available is methylprednisolone acetate injection 80 mg/ml. How many mLs should the nurse administer?

1.5 mL

a nurse is preparing to administer methylprednisolone acetate 60 mg im every week. Available is methylprednisolone acetate injection 40 mg/ml. how many ml should the nurse administer per dose?

1.5 mL

A nurse is preparing to administer doxepin 75 mg PO at bedtime. Available is doxepin 50 mg tablets. How many tablets should the nurse administer?

1.5 tablets

A nurse is preparing to administer furosemide 60 mg PO daily. Available is furosemide 40 mg tablets. How many tablets should the nurse administer daily?

1.5 tablets

A nurse is preparing to administer acetaminophen 325 mg PO every 6 hr. Available is acetaminophen oral suspension 160 mg/5 mL. How many mL should the nurse administer per dose?

10 mL

A nurse is preparing to administer erythromycin ethylsuccinate 800 mg PO every 12 hr. Available is erythromycin ethylsuccinate suspension 400mg/5mL. How many mL should the nurse administer per dose?

10 mL

A nurse is preparing to administer phenytoin (Dilantin) 15 mg/kg/day PO divided in equal doses every 8 hr to a client who weighs 120 lb. Available is phenytoin suspension 125 mg/5 mL. How many mL should the nurse administer per dose?

10.9 mL

The nurse preparing to transfer to a pediatric hospital would expect to administer medications to children from birth to age _______ years.

16

A nurse is preparing to administer acyclovir 5 mg/kg IV bolus every 8 hr to a client who weighs 75 lb. How many mg should the nurse administer per dose?

170.5

A nurse is preparing to administer codeine 30 mg PO every 4 hr PRN for pain. The client last received the medication at 1 p.m. The earliest the nurse should administer the next dose is at which of the following times? 1400 1500 1600 1700

1700

A nurse is preparing to administer potassium chloride powder 20 mEq PO in 6 oz of water. How many mL of water should the nurse administer?

180 mL

The nurse uses a newly admitted​ client's armband barcode to identify the client prior to administering medications. The nurse should use at least ___ other means of identifying this patient

2

The nurse who is writing a risk nursing diagnosis will write a _____ part statement.

2

A nurse is preparing to administer gabapentin 1,800 mg per day divided in three equal doses. The amount available is gabapentin 300 mg capsules. How many capsules should the nurse administer per dose?

2 capsules

A nurse is preparing to administer lithium 600 mg PO every 8 hr. Available is lithium carbonate 300 mg capsules. How many capsules should the nurse administer?

2 capsules

A nurse is preparing to administer triazolam 0.25 mg PO. Available is triazolam 0.125 mg tablets. How many tablets should the nurse administer per dose?

2 capsules

a nurse is preparing to administer phenytoin 100 mg IV bolus every 8hr. Available is phenytoin injection 50 mg/ml. How many mL should the nurse administer per dose?

2 mL

A nurse is preparing to administer zolpidem 10 mg PO at bedtime. Available is zolpidem 5 mg tablets. How many tablets should the nurse administer per dose?

2 tablets

A nurse is preparing to administer magnesium gluconate 270 mg PO. Available is magnesium gluconate liquid 54 mg/5 mL. How many mL should the nurse administer?

25 mL

A nurse working in the emergency department is told that the smallpox virus was aerosolized at a parade attended by thousands of people. The nurse explains to the student nurse that the vaccine for this virus provides a high level of protection if given within days of exposure.

3

a nurse is preparing to administer furosemide 30 mg iv bolus stat. Available is furosemide injection 10 mg/mL. How many mL should the nurse administer?

3 mL

A nurse is preparing to administer levothyroxine 0.075 mg PO to a client. Available is levothyroxine 25 mcg tablets. How many tablets should the nurse administer?

3 tablets

A nurse is preparing to administer amoxicillin 25 mg/kg/day PO divided in equal doses every 8 hr to a preschooler who weights 43 lb. Avaliable is amoxicillin suspension 250 mg/5 ml. How many ml should the nurse administer per dose?

3.3 mL

A nurse is preparing to administer cefaclor 40 mg/kg/day PO divided in equal doses 8 hr to a child who weighs 48;n. Available is cefaclor suspension 375 mg/5 ml. How many ml should the nurse administer per dose.

3.9

A nurse is converting a toddler's weight from lb to kg. if the toddler weighs 17 lb 9 oz, what is the toddler's weight in kg?

8

A nurse is preparing to administer fluoxetine 35 mg PO daily. Available is fluoxetine solution 20 mg/5 mL. How many mL should the nurse administer daily?

8.8 mL

A nurse is convertive a todder's weight from lb to kg. If the toddler weighs 20 lb 8 oz, what is the toddler's weight in kg

9.3 kg

A woman who wishes to become pregnant is concerned about the drugs she must take in order to treat a serious medical condition. The nurse reviewing the drug list would be most concerned about which kind of​ drug? A. A drug with a long​ half-life. B. A drug with high​ protein-binding ability. C. A drug with no active metabolites. D. A drug taken only when needed.

A. A drug with a long​ half-life.

The nurse is working hard to prevent medication errors. What interventions will assist the nurse in preventing most​ errors? Select all that apply. A. Always check the​ client's identification band prior to administration of medications. B. Open all of the medications immediately prior to administration. C. Tell healthcare providers that verbal orders will not be accepted. D. Record the medication on the medication administration record​ (MAR) immediately prior to administration. E. Validate all orders with another nurse prior to administration of medications.

A. Always check the​ client's identification band prior to administration of medications. B. Open all of the medications immediately prior to administration. C. Tell healthcare providers that verbal orders will not be accepted.

Which patient statements would the nurse evaluate as negatively affecting access to​ healthcare? Select all that apply. A. An older adult​ says, "If I tell my doctor that I fall​ frequently, I might have to go to a nursing​ home." B. A young adult​ says, "A benefit of my new job is that it pays for my​ health, dental, and vision​ insurance." C. A client being discharged​ says, "Can you call my prescriptions in to the​ pharmacy?" D. The​ client's new insurance plan requires a​ $10 copay for office visits. E. A client asks to be discharged before noon as he has a​ 3-hour drive home.

A. An older adult​ says, "If I tell my doctor that I fall​ frequently, I might have to go to a nursing​ home." D. The​ client's new insurance plan requires a​ $10 copay for office visits. E. A client asks to be discharged before noon as he has a​ 3-hour drive home.

A nurse is planning a presentation regarding anthrax. What should the nurse​ include? Select all that apply. A. Anthrax can spread easily. B. There are three basic types of anthrax. C. Anthrax is killed by heat. D. The main area affected by inhalation anthrax is the lung periphery. E. Anthrax causes the release of multiple​ toxins, each affecting a different part of the body.

A. Anthrax can spread easily. B. There are three basic types of anthrax.

Which client would be most likely to seek complementary and alternative therapies as a treatment​ modality? Select all that apply. A. A​ 48-year-old male who wants to boost his immune system. B. A​ 22-year-old male athlete who is preparing for the Olympics. C. A​ 50-year-old female going through menopause. D. A​ 58-year-old male with prostate problems. E. None of these clients would be wise to seek these therapies.

A. A​ 48-year-old male who wants to boost his immune system. B. A​ 22-year-old male athlete who is preparing for the Olympics. C. A​ 50-year-old female going through menopause. D. A​ 58-year-old male with prostate problems.

The nurse is doing a holistic assessment on a client prior to starting antihypertensive medication. What should the nurse include in the​ assessment? Select all that apply. A. Blood pressure B. The biologic cause of the hypertension C. Mood D. Level of education E. Belief in a higher power

A. Blood pressure C. Mood D. Level of education E. Belief in a higher power

The client asks if there are any supplements that would help improve or preserve his memory. Which supplements would the nurse​ discuss? Select all that apply. A. Carnitine B. Coenzyme Q10 C. Glucosamine D. DHEA E. Selenium

A. Carnitine D. DHEA

A client is admitted to the emergency department with suspected exposure to anthrax. The nurse anticipates the healthcare provider will prescribe which​ medication? A. Ciprofloxacin. B. Antiviral agents. C. Anthrax vaccination. D. Atropine.

A. Ciprofloxacin.

The client has been exposed to anthrax. The nurse anticipates administering which of the following​ drugs? A. Ciprofloxacin​ (Cipro) and doxycycline​ (Vibramycin). B. Tetracycline​ (Sumycin) and erythromycin​ (Erythrocin). C. Penicillin​ (Bicillin LA) and vancomycin​ (Vancocin). D. Ampicillin​ (Principen) and cefepime​ (Maxipime).

A. Ciprofloxacin​ (Cipro) and doxycycline​ (Vibramycin).

What is the most significant role for nurses as defined by state nurse practice acts and by regulating bodies such as The Joint​ Commission? A. Client teaching B. Discharging clients C. Prescribing medication D. Ordering lab tests

A. Client teaching

The nurse in charge of a clinical study welcomes the participants in an open forum. One client is surprised that there are men and women from several ethnic groups. The nurse explains that in the​ past, ethnic variables were largely unknown or ignored for what​ reasons? Select all that apply. A. Clinical trials failed to include ethnically diverse subjects. B. Clinical trials comprised mostly Caucasian males. C. Little attention was focused on identifying the different effects drugs had on various ethnic groups. D. Research proved there were no differences among ethnic groups. E. The large majority of clinical trials included Caucasian females.

A. Clinical trials failed to include ethnically diverse subjects. B. Clinical trials comprised mostly Caucasian males. C. Little attention was focused on identifying the different effects drugs had on various ethnic groups.

A client is diagnosed with cancer. The healthcare provider has recommended​ chemotherapy, which would likely save the​ client's life. The client tells the​ nurse, "This is punishment from God for sins I have​ committed; some women at my church say​ so."? With permission from the​ client, what would the nurse do​ first? A. Contact the​ client's minister to discuss the​ client's perspective about cancer. B. Involve a hospital minister to discuss the​ client's perspective about cancer. C. Plan to bring the case before the​ hospital's board of ethics. D. Meet with family members to discuss the​ client's perspective about cancer.

A. Contact the​ client's minister to discuss the​ client's perspective about cancer.

Following a bioterrorism​ attack, the nurse finds that the victims are suffering from​ small, black lesions on their forearms. The nurse collaborates with the healthcare provider to treat exposure to which​ agent? A. Cutaneous anthrax B. Hydrogen cyanide C. Phosgene gas D. Gastrointestinal anthrax

A. Cutaneous anthrax

The African American client had a myocardial infarction and is receiving atorvastatin​ (Lipitor). The nurse assesses the​ client's diet to be very high in fat. What is the best plan by the nurse to improve the​ client's diet and reduce the risk for additional​ medications? A. Discuss his diet with whomever prepares meals for his family. B. Obtain a consult for dietary services so a dietician can teach the client about​ low-fat diets. C. Ask an African American nurse to speak to him about a​ low-fat diet. D. Give the client information specific to African Americans about​ low-fat diets.

A. Discuss his diet with whomever prepares meals for his family.

A home health​ nurse's patient caseload is ethnically diverse. Which interventions show understanding of cultural​ variables? Select all that apply. A. Discussing cultural preferences for herbs and spices and possible alternatives when drug interactions are possible. B. Assessing the​ client's response to acupuncture for pain. C. Discussing the​ client's beliefs regarding treatment. D. Notifying the healthcare provider of the​ client's intentions to consult with a medicine man for spiritual guidance. E. Removing the​ client's collection of herbs to decrease the risk of an adverse effect when taken with Western medicine

A. Discussing cultural preferences for herbs and spices and possible alternatives when drug interactions are possible. B. Assessing the​ client's response to acupuncture for pain. C. Discussing the​ client's beliefs regarding treatment.

Which information is essential for the nurse to collect when reviewing a​ client's medication​ list? Select all that apply. A. Drug names B. Drug manufacturer C. Dosage being taken D. Frequency of administration E. When last refill was obtained

A. Drug names C. Dosage being taken D. Frequency of administration

The nurse understands that drug therapy is postponed until after pregnancy and lactation when possible. Which of the following acute and chronic conditions may be managed with drugs during​ pregnancy? Select all that apply. A. Epilepsy B. Serious cystic acne C. Sexually transmitted infections D. Gestational diabetes E. Hypertension

A. Epilepsy C. Sexually transmitted infections D. Gestational diabetes E. Hypertension

A client with inflammatory arthritis asks about adding an​ anti-inflammatory herb to the medication regimen to decrease the use of prescription drugs. The nurse suggests the client discuss the efficacy of which herbs with the healthcare​ provider? Select all that apply. A. Evening primrose B. St.​ John's wort C. Ginger D. Ginseng E. Bilberry

A. Evening primrose B. St.​ John's wort C. Ginger

A​ client's genetic testing indicates the presence of a genetic polymorphism of the CYP 450 enzyme. The nurse expects this difference to be one of which​ classifications? Select all that apply. A. Extensive B. Poor C. Normal D. Ultrarapid E. Inefficient

A. Extensive B. Poor D. Ultrarapid

The nurse makes a medication​ error, but the client is not harmed. The​ client's family asks the nurse manager what is considered a medication error. How should the nurse manager​ respond? Select all that apply. A. Failure to follow healthcare​ provider's orders. B. Failure to give the right medication. C. Failure to give a medication at the ordered time. D. Failure to call the pharmacy and report that the medication has been given. E. Failure to give the right dose of the medication.

A. Failure to follow healthcare​ provider's orders. B. Failure to give the right medication. C. Failure to give a medication at the ordered time. E. Failure to give the right dose of the medication.

A client with a family history of heart disease asks the nurse to recommend herbs that decrease the risk for heart disease. The nurse suggests the client discuss which herbs with the healthcare​ provider? Select all that apply. A. Flaxseed B. Green tea C. Soy D. Milk thistle E. Acai

A. Flaxseed B. Green tea C. Soy

A client who is considering taking a dietary supplement recommended by the nurse asks how to determine if the supplement is safe. The nurse explains that the Dietary Supplement and Nonprescription Drug Consumer Protection Act of 2007 requires that companies marketing herbal and dietary supplements do which of the​ following? Select all that apply. A. Include their contact information on the label so consumers can report adverse effects. B. Notify the Food and Drug Administration​ (FDA) within 15 days of receiving a report of an adverse effect. C. Keep records of adverse effects for at least 6 years and make these records available for inspection by the Food and Drug Administration​ (FDA). D. Demonstrate product effectiveness prior to release of the product to the public. E. State which conditions the product is intended to cure or prevent.

A. Include their contact information on the label so consumers can report adverse effects. B. Notify the Food and Drug Administration​ (FDA) within 15 days of receiving a report of an adverse effect. C. Keep records of adverse effects for at least 6 years and make these records available for inspection by the Food and Drug Administration​ (FDA).

A nurse working in an obstetric practice should consider which fact when discussing medication use with pregnant​ clients? A. Inhaled drugs may be absorbed to a greater extent. B. Oral drug absorption rate is lowered. C. Drug excretion rates are lowered. D. Drugs should not be used during pregnancy.

A. Inhaled drugs may be absorbed to a greater extent.

A student nurse asks a nurse what is included in the Strategic National Stockpile​ (SNS). The nurse explains that the SNS includes which of the following​ supplies? Select all that apply. A. Intravenous administration equipment B. Antibiotics C. ​Life-support medications D. Chemical antidotes E. Hospital beds

A. Intravenous administration equipment B. Antibiotics C. ​Life-support medications D. Chemical antidotes

The nurse works for the Centers for Disease Control and Prevention​ (CDC). In planning for a bioterrorist​ attack, what will the best plan of the nurse​ include? Select all that apply. A. Learn the signs and symptoms of chemical and biological agents. B. Obtain a listing of health and law enforcement contacts. C. Assist in the stockpiling of medications. D. Obtain current knowledge of emergency management. E. Assist in triage at local hospitals.

A. Learn the signs and symptoms of chemical and biological agents. B. Obtain a listing of health and law enforcement contacts. D. Obtain current knowledge of emergency management.

A community health nurse is preparing a teaching plan regarding medications and their potential adverse effects for a new parent class. The nurse should encourage parents to do which of the​ following? Select all that apply. A. Maintain a list of current medications for each child. B. Be aware of each​ child's medication allergies. C. Know what the​ child's prescribed medication is​ for, how it should be​ administered, and when to expect the child to feel better. D. Be aware that any leftover medication should be appropriately disposed​ of, not saved for future use. E. Read the prescription label for any foods the child should avoid while taking the medication and for possible adverse effects to watch out for.

A. Maintain a list of current medications for each child. B. Be aware of each​ child's medication allergies. C. Know what the​ child's prescribed medication is​ for, how it should be​ administered, and when to expect the child to feel better. D. Be aware that any leftover medication should be appropriately disposed​ of, not saved for future use.

A nurse is preparing care for a newly admitted client with diabetes. Which information would be critical for the nurse to​ assess? Select all that apply. A. Medical history B. Current lab results C. Medication allergies D. Use of dietary supplements E. Number of previous hospitalizations

A. Medical history B. Current lab results C. Medication allergies D. Use of dietary supplements

A nurse on the​ medical-surgical unit is caring for several very ill clients. One client​ says, "I was supposed to get my medications an hour​ ago." The nurse recognizes that medication errors can have what​ impact? Select all that apply. A. Medication errors can potentially extend the​ client's length of hospital stay. B. Medication errors can result in expensive legal costs to the facility. C. Medication errors can damage the​ facility's reputation. D. Medication errors can be physically devastating to nurse and client. E. Medication errors cause preventable deaths during hospitalizations.

A. Medication errors can potentially extend the​ client's length of hospital stay. B. Medication errors can result in expensive legal costs to the facility. C. Medication errors can damage the​ facility's reputation. E. Medication errors cause preventable deaths during hospitalizations.

The nurse preparing a teaching plan for a postpartum mother. The nurse plans to include which topics regarding​ breastfeeding? Select all that apply. A. Most medications are secreted into breast milk. B. Dietary supplements should be avoided. C. Herbal products are secreted into breast milk and have the potential to cause harm to the infant. D. All tobacco products should be avoided. E. Most drugs are safe to take right after breastfeeding because they will clear the bloodstream prior to the next feeding.

A. Most medications are secreted into breast milk. B. Dietary supplements should be avoided. C. Herbal products are secreted into breast milk and have the potential to cause harm to the infant. D. All tobacco products should be avoided.

A nurse is teaching a student nurse about common treatments for victims of bioterrorism. The nurse tells the student that atropine would be most useful for a victim of bioterrorism exposed to which of the​ following? A. Nerve gas B. Bacterial agents C. Viral agents D. Ionizing radiation

A. Nerve gas

The nurse administers an evening medication to the client in the morning. What is the​ nurse's best initial course of action at this​ time? A. Notify the healthcare provider about the error. B. Change the medication administration time to the morning. C. Tell the evening nurse to hold the evening dose just for tonight. D. Document the incident in the​ client's health record.

A. Notify the healthcare provider about the error.

The nurse is caring for several clients. Which clients have a psychosocial history that may affect their​ outcome? Select all that apply. A. Older adult who recently suffered a​ stroke, has an unsteady​ gait, and lives in a​ two-story home B. ​Middle-aged client with Down syndrome living in a group home C. Recently divorced mother of three children with breast cancer D. ​Sixteen-year-old requesting birth control without parental consent E. ​Seven-year-old with asthma in a foster care home

A. Older adult who recently suffered a​ stroke, has an unsteady​ gait, and lives in a​ two-story home B. ​Middle-aged client with Down syndrome living in a group home C. Recently divorced mother of three children with breast cancer

The nurse has been hired to work in the risk management office of a hospital. What situations would the nurse expect to be included in this​ job? Select all that apply. A. Participating in the investigation of a sentinel medication event B. Using the computer to track data C. Working with staff nurses to identify work flow problems D. Meeting with the nurse executive to identify nurses who are prone to medication errors E. Participating in a committee who will recommend changes to the policy and procedures regarding medication administration.

A. Participating in the investigation of a sentinel medication event B. Using the computer to track data C. Working with staff nurses to identify work flow problems E. Participating in a committee who will recommend changes to the policy and procedures regarding medication administration.

During​ evaluation, the​ nurse, client, and healthcare provider determine that the goals of antibiotic therapy have not been met. What actions are​ indicated? Select all that apply. A. Review the dosage of the medication B. Consider checking serum drug levels C. Discard the idea that the infection is treatable D. Consider prolonging therapy E. Consider using a different antibiotic

A. Review the dosage of the medication B. Consider checking serum drug levels D. Consider prolonging therapy E. Consider using a different antibiotic

A nurse is admitting a client to the emergency department who has overdosed on a benzodiazepine. The nurse should anticipate the healthcare provider will prescribe which of the following​ drugs? A. Romazicon B. Acetylcysteine C. Acetaminophen D. Digibind

A. Romazicon

A postmenopausal client with a history of stroke is told she cannot take female hormones because of the risk for future thromboembolic events. The client asks about nonpharmacologic ways to manage her symptoms. Which herbs does the nurse suggest as​ alternatives? Select all that apply. A. Soy B. Black cohosh C. Evening primrose D. Saw palmetto E. Acai

A. Soy B. Black cohosh C. Evening primrose

A nurse is providing care to a client who wears bilateral hearing aids. Which nursing interventions are​ indicated? Select all that apply. A. Speak a bit slower than normal. B. Speak more loudly than normal. C. Ensure that verbal and nonverbal communication is congruent. D. Allow extra time for communication. E. Use bright lighting in the room.

A. Speak a bit slower than normal. C. Ensure that verbal and nonverbal communication is congruent. D. Allow extra time for communication.

A nurse has been asked to present health promotion information at a community clinic whose clients are primarily​ non-Hispanic Black women. After review of health disparity and inequality statistics the nurse chooses to include information about which​ disorders? Select all that apply. A. Stroke B. Coronary artery disease C. Diabetes D. Illicit drug use E. Prenatal care

A. Stroke B. Coronary artery disease C. Diabetes E. Prenatal care

The adolescent is supposed to go to the school nurse at​ 12:00 to receive his medication for​ attention-deficit/ hyperactivity disorder​ (ADHD). He often does not go for the medication. What best describes the​ nurse's understanding of this​ situation? A. The adolescent is embarrassed in front of his peers. B. The adolescent forgets that he is supposed to take the medication. C. The adolescent has made a conscious decision not to take the medication. D. The adolescent does not understand the need for the medication.

A. The adolescent is embarrassed in front of his peers.

A nurse is caring for a client that was exposed to radiation. The client asks the nurse what factors increase the amount of radiation that a person is exposed to. What should the nurse include in responding to the​ client's question? Select all that apply. A. The amount of exposure. B. The​ long-lasting effects. C. The amount of cellular death. D. The gender of the client exposed. E. The amount of naloxone​ (Narcan) ingested after exposure.

A. The amount of exposure. B. The​ long-lasting effects. C. The amount of cellular death.

A client presents with hypotension and bradycardia. The client indicates that one of her healthcare providers recently prescribed three new medications to her current list of 10 medications per day. Based on this​ information, which statement would be the most​ accurate? A. The client is experiencing adverse reactions as a result of polypharmacy. B. The client is having an allergic reaction to one of the new medications. C. The client is not in compliance with her prescribed medications. D. The client is experiencing an adverse effect that will go away in time

A. The client is experiencing adverse reactions as a result of polypharmacy.

For which client would the nurse expect the healthcare provider to continue prescribed medications during​ pregnancy? Select all that apply. A. The client recently diagnosed with gonorrhea B. The client with a history of frequent asthma attacks C. The client with hypertension D. The client with frequent insomnia E. The client with a family history of stroke

A. The client recently diagnosed with gonorrhea B. The client with a history of frequent asthma attacks C. The client with hypertension

The nurse is preparing a teaching plan for an older client who is taking multiple medications. Which principles should the nurse keep in mind during the planning​ phase? Select all that apply. A. The client should have all prescriptions filled at the same pharmacy. B. The client should keep a list of all medications for easy accessibility. C. Older clients often take multiple drugs which is a common cause of medication errors. D. Polypharmacy is unique to older clients and is the most common cause of medication errors. E. The client should be aware of each prescribed​ medication, the​ dose, and possible side effects.

A. The client should have all prescriptions filled at the same pharmacy. B. The client should keep a list of all medications for easy accessibility. C. Older clients often take multiple drugs which is a common cause of medication errors. E. The client should be aware of each prescribed​ medication, the​ dose, and possible side effects.

The nursing instructor teaches the student nurses about how medication errors can occur. What information will the nursing instructor include in the​ presentation? Select all that apply. A. The nurse miscalculates the medication dose. B. The nurse does not check the​ client's identification band. C. The nurse does not validate an order with the healthcare provider. D. The nurse misinterprets a healthcare​ provider's order. E. The nurse administers the incorrect drug.

A. The nurse miscalculates the medication dose. B. The nurse does not check the​ client's identification band. D. The nurse misinterprets a healthcare​ provider's order. E. The nurse administers the incorrect drug.

A nurse is teaching a community group about holistic medicine. Which information should the nurse include about Western​ medicine? Select all that apply. A. Western medicine focuses on the cause of disease. B. Western medicine is not compatible with holistic medicine. C. Western medicine is not compatible with holistic medicine. D. Disease is often viewed as a malfunction in a specific body system. E. Western medicine may focus on disease treatment.

A. Western medicine focuses on the cause of disease. D. Disease is often viewed as a malfunction in a specific body system. E. Western medicine may focus on disease treatment.

The client​ says, "I am convinced that the​ mind-body interventions I am using are improving my​ health." The nurse would ask additional questions about which complementary and alternative therapies that belong to this​ category? Select all that apply. A. Yoga B. Massage C. Dance D. Herbal therapies E. Biofeedback

A. Yoga B. Massage C. Dance E. Biofeedback

The nurse assesses the client with diabetes mellitus prior to administering medications. Which questions are important to ask the​ client? Select all that apply. A. ​"Are you allergic to any​ medications?" B. ​"Are you taking any herbal or​ over-the-counter (OTC)​ medications?" C. ​"How difficult is it for you to maintain your ideal body​ weight?" D. ​"Will you please tell me about the kind of diet you​ follow?" E. ​"What other medications are you currently​ taking?"

A. ​"Are you allergic to any​ medications?" B. ​"Are you taking any herbal or​ over-the-counter (OTC)​ medications?" D. ​"Will you please tell me about the kind of diet you​ follow?" E. ​"What other medications are you currently​ taking?"

The public health nurse notices that several clients in the hypertension clinic have poorly controlled​ hypertension, even though they have been prescribed appropriate antihypertensive drugs. Which question will best enable the nurse to assess these​ clients? A. ​"Can you afford the high blood pressure​ medication?" B. ​"Does your religion allow the use of high blood pressure​ medication?" C. ​"Do you think your high blood pressure is a​ problem?" D. ​"Does your culture use herbs to treat high blood​ pressure?"

A. ​"Can you afford the high blood pressure​ medication?"

The nurse follows the nursing process when conducting medication education about insulin. What will the nurse ask the client to evaluate the​ client's knowledge of​ insulin? A. ​"Can you tell me four points you remember about how to take your​ insulin?" B. ​"Can you recognize when you are experiencing​ hypoglycemia?" C. ​"What questions do you have about​ insulin?" D. ​"Is your abdomen the best place to inject​ insulin?"

A. ​"Can you tell me four points you remember about how to take your​ insulin?"

A nurse who is presenting community education​ states, "Drugs are among our most powerful weapons for emergency​ preparedness." What additional​ statements, made by the​ nurse, would support this​ statement? Select all that apply. A. ​"Drugs help us treat the diseases that could be caused by​ bioterrorism." B. ​"We can use drugs to neutralize our​ enemies." C. ​"If we control the manufacture and distribution of​ drugs, we will control global​ politics." D. ​"Without drugs, we might be quickly overwhelmed by a bioterrorist​ attack." E. ​"Drugs can be used in chemical or nuclear​ attacks."

A. ​"Drugs help us treat the diseases that could be caused by​ bioterrorism." D. ​"Without drugs, we might be quickly overwhelmed by a bioterrorist​ attack." E. ​"Drugs can be used in chemical or nuclear​ attacks."

An adolescent client comes to the school nurse with complaints of vague abdominal pain. What statement by the student would help to confirm the​ nurse's suspicion that the adolescent has body image​ concerns? A. ​"Everyone makes a big deal about what I​ eat, so​ don't ask." B. ​"I just​ can't seem to get along with my​ parents." C. ​"My periods are irregular. Should I see a​ physician?" D. ​"I have been sexually active with my​ boyfriend."

A. ​"Everyone makes a big deal about what I​ eat, so​ don't ask."

A client asks the nurse why a medication prescribed by the provider​ "didn't do anything at​ all." Which statement by the nurse accurately describes how genetics influence drug​ action? Select all that apply. A. ​"Genetic differences can result in significant differences in how each​ client's body handles the same​ medication." B. ​"Genetic differences can cause mutations in​ enzymes, changing the way they function. This can alter how the body metabolizes and excretes​ drugs." C. ​"Because of genetic​ differences, medication may accumulate to toxic levels in one client while in another client may be inactivated before it can have a therapeutic​ effect." D. ​"Genetic differences can be expressed as an alteration in the structure of an​ enzyme, which can cause a defective receptor and an allergic response to​ drugs." E. ​"Genetic differences in clients who are biracial result in an allergic response to​ medications."

A. ​"Genetic differences can result in significant differences in how each​ client's body handles the same​ medication." B. ​"Genetic differences can cause mutations in​ enzymes, changing the way they function. This can alter how the body metabolizes and excretes​ drugs." C. ​"Because of genetic​ differences, medication may accumulate to toxic levels in one client while in another client may be inactivated before it can have a therapeutic​ effect."

The nurse is conducting a holistic assessment of a client with alcoholism. What are the important questions the nurse would​ ask? Select all that apply. A. ​"How is drinking alcohol viewed by your​ culture?" B. ​"Have you ever attended Alcoholics Anonymous​ meetings?" C. ​"Did you see your parents drinking alcohol when you were growing​ up?" D. ​"Have you been in alcohol rehabilitation before​ now?" E. ​"What blood relatives of yours are addicted to​ alcohol?"

A. ​"How is drinking alcohol viewed by your​ culture?" C. ​"Did you see your parents drinking alcohol when you were growing​ up?" E. ​"What blood relatives of yours are addicted to​ alcohol?"

The nurse conducts a seminar in a local community center on how citizens can be affected by radiation from a nuclear attack. The nurse determines that the education is effective when the clients make which​ statement? A. ​"I am at risk to develop leukemia as a result of radiation​ exposure." B. ​"I need to take at least four showers every day or I will develop skin​ ulcers." C. ​"I can protect myself from cancers by taking potassium iodide​ (KI)." D. ​"I need to stay inside my house for at least 2 days after the attack to be​ safe."

A. ​"I am at risk to develop leukemia as a result of radiation​ exposure."

The nurse is teaching clients in a senior center about complementary and alternative therapies. Which statement by a client indicates understanding of the information the nurse has​ presented? Select all that apply. A. ​"I see now that​ 'natural' doesn't really mean​ 'safe,' and I should ask my healthcare provider about any supplements I want to​ take." B. ​"I didn't realize some herbal supplements may be just as strong as a prescription drug. I will need to discuss any supplements I want to try with my​ doctor." C. ​"I understand now that when I choose to use a natural product over a prescription​ medication, I could be delaying my​ recovery." D. ​"It's good to know that herbs contain ingredients that are as powerful as a prescribed medication and are an option for safe and effective​ therapy." E. ​"It's good to know that herbs contain ingredients that are as powerful as a prescribed medication and are an option for safe and effective​ therapy."

A. ​"I see now that​ 'natural' doesn't really mean​ 'safe,' and I should ask my healthcare provider about any supplements I want to​ take." B. ​"I didn't realize some herbal supplements may be just as strong as a prescription drug. I will need to discuss any supplements I want to try with my​ doctor." C. ​"I understand now that when I choose to use a natural product over a prescription​ medication, I could be delaying my​ recovery."

A nurse has provided discharge medication instructions to the parents of a child being released from the emergency department. The nurse evaluates that learning has occurred when the parents make which​ statements? Select all that apply. A. ​"I should give this antibiotic as prescribed until the bottle is​ empty." B. ​"I should use a spoon to give this​ medication." C. ​"I will keep this medication in the refrigerator as the label​ directs." D. ​"If my child develops any adverse​ effects, I will discard the rest of the​ medicine." E. ​"This antibiotic should help to clear my​ child's infection."

A. ​"I should give this antibiotic as prescribed until the bottle is​ empty." C. ​"I will keep this medication in the refrigerator as the label​ directs." E. ​"This antibiotic should help to clear my​ child's infection."

The nurse in the emergency department is caring for several clients from diverse cultures. Which statement shows the​ nurse's ability to provide culturally competent​ care? Select all that apply. A. ​"I understand your religion prohibits blood transfusions. Would you consider nonblood​ alternatives?" B. ​"I just want to make sure you and your spouse understand the risks as you consider the​ options." C. ​"I don't really understand why you are afraid to take the medication. Do you have any questions I can answer to alleviate your​ fear?" D. ​"I really​ don't understand why you​ won't consider an abortion. Your admission papers say you are an​ atheist." E. ​"I'm not quite sure why the healthcare provider is giving you these prescriptions. You​ didn't get them filled the last time you were​ here."

A. ​"I understand your religion prohibits blood transfusions. Would you consider nonblood​ alternatives?" B. ​"I just want to make sure you and your spouse understand the risks as you consider the​ options." C. ​"I don't really understand why you are afraid to take the medication. Do you have any questions I can answer to alleviate your​ fear?"

A client returns to the clinic for​ follow-up after taking a newly prescribed medication for a month. The nurse recognizes medication teaching was successful when the client makes which​ statement? Select all that apply. A. ​"I've been taking my medication on an empty stomach like the prescription label said​ to." B. ​"I take my medication first thing in the​ morning, just like you​ said." C. ​"I have been able to decrease my medication to every other day and that saves me some​ money." D. ​"I switched all my medications to one pharmacy like you​ suggested." E. ​"Did you say I need to take this medication with water or​ milk?"

A. ​"I've been taking my medication on an empty stomach like the prescription label said​ to." B. ​"I take my medication first thing in the​ morning, just like you​ said." D. ​"I switched all my medications to one pharmacy like you​ suggested."

A new nurse on the orthopedic floor makes a medication error. Which statements by the nurse manager foster a safe environment in which nurses will report medication​ errors? Select all that apply. A. ​"Many of us have made a medication error in our careers. The most important issue is to identify why the error​ occurred." B. ​"I know you could not feel any worse than you already do. We need to discuss how this error happened and how we can prevent it from happening​ again." C. ​"It's really good that your client is OK and did not suffer any harmful effects of this error. We should discuss why this error occurred and how it can be prevented in the​ future." D. ​"Because you are a new​ nurse, we should sit down and discuss the procedure you followed to see what you could have done to prevent this​ error." E. ​"We need to sit down as soon as possible and write up an incident report describing everything you did incorrectly that caused this​ error."

A. ​"Many of us have made a medication error in our careers. The most important issue is to identify why the error​ occurred." B. ​"I know you could not feel any worse than you already do. We need to discuss how this error happened and how we can prevent it from happening​ again." C. ​"It's really good that your client is OK and did not suffer any harmful effects of this error. We should discuss why this error occurred and how it can be prevented in the​ future." D. ​"Because you are a new​ nurse, we should sit down and discuss the procedure you followed to see what you could have done to prevent this​ error."

The nurse is planning care for a pregnant client prone to substance abuse. When the client​ states, "My baby​ isn't getting my​ drugs, I​ am," how does the nurse​ respond? Select all that apply. A. ​"Most illicit drugs will cross the placenta and hurt the​ baby." B. ​"Drugs may work longer in your​ baby." C. ​"Some drugs result in your baby not growing​ enough." D. ​"You are correct. You are far enough along in your pregnancy that drugs will not harm your​ baby." E. ​"If you continue to take​ drugs, it will make you have a large​ infant."

A. ​"Most illicit drugs will cross the placenta and hurt the​ baby." B. ​"Drugs may work longer in your​ baby." C. ​"Some drugs result in your baby not growing​ enough."

The nurse is teaching a class on dietary supplements. The nurse determines that learning has occurred when the patients make which​ statement? A. ​"Products can make claims based on body structure and function like​ 'promotes healthy urinary​ tract.'" B. ​"The Food and Drug Administration​ (FDA) is not involved with the approval of the dietary supplement and cannot remove it from the​ market." C. ​"The manufacturer of the dietary supplement has the burden of proof for the safety of the​ supplement." D. ​"Dietary supplements must go through rigorous testing prior to being marketed by the​ manufacturer."

A. ​"Products can make claims based on body structure and function like​ 'promotes healthy urinary​ tract.'"

The nurse plans to teach clients about the difference between herbal preparations and prescription medications. Which statement describes the best plan by the​ nurse? A. ​"The active ingredients in an​ herb, unlike prescription​ drugs, may be present in just one part or many​ parts." B. ​"Herbs are considered to be safer than prescription medications as they are​ natural." C. ​"Most herbs, like prescription​ drugs, are standardized and regulated by the Food and Drug Administration​ (FDA)." D. ​"Herbs, like prescription​ drugs, usually only contain one active​ ingredient."

A. ​"The active ingredients in an​ herb, unlike prescription​ drugs, may be present in just one part or many​ parts."

The nurse is teaching the importance of drugs for emergency preparedness to local firemen. The nurse determines that learning has occurred when the firemen make which​ statements? Select all that apply. A. ​"The vendor-managed inventory​ (VMI) package can reach any community within 24 to 36​ hours." B. ​"Our local hospital is supposed to be stockpiling​ antibiotics." C. ​"The push package can reach any community within 12 hours of an​ attack." D. ​"The Strategic National Stockpile is located at the Centers for Disease Control and Prevention​ (CDC) in​ Atlanta." E. ​"Our country's drug stockpile is managed by the Centers for Disease Control and Prevention​ (CDC)."

A. ​"The vendor-managed inventory​ (VMI) package can reach any community within 24 to 36​ hours." Select all that apply. C. ​"The push package can reach any community within 12 hours of an​ attack." E. ​"Our country's drug stockpile is managed by the Centers for Disease Control and Prevention​ (CDC)."

The risk management department is using a​ root-cause analysis to improve a nursing​ unit's medication administration accuracy. What questions will be used to develop this​ tool? Select all that apply. A. ​"What kind of errors are​ occurring?" B. ​"What is the current medication administration accuracy​ rate?" C. ​"How do the unit nurses rank in the number of errors​ committed?" D. ​"What do the nurses think can be done to prevent errors from​ continuing?" E. ​"What is the impact of changes made to improve​ accuracy?"

A. ​"What kind of errors are​ occurring?" B. ​"What is the current medication administration accuracy​ rate?" D. ​"What do the nurses think can be done to prevent errors from​ continuing?" E. ​"What is the impact of changes made to improve​ accuracy?"

The client is admitted to the hospital with sepsis following an elective abortion. The healthcare provider orders antibiotics that the client refuses​ stating, "I​ don't deserve​ them". What is the best response by the​ nurse? A. ​"You have a serious infection and really need the​ drug." B. ​"I'll call your healthcare provider and let him know about your​ decision." C. ​"It seems you think you should be punished because you had an​ abortion" D. ​"I think you need to do what is best for​ you."

A. ​"You have a serious infection and really need the​ drug."

A nurse administering medications to a variety of patients on a​ medical-surgical floor recognizes that which clients may need additional education about medication​ adherence? Select all that apply. A. ​Fifty-year-old recently remarried male taking antihypertensive medication B. ​Thirty-four-year-old female with family history of blood clots taking an estrogen oral contraceptive C. ​Thirty-eight-year-old male recently started on an antidepressant D. ​Twenty-eight-year-old female started on acne medication known to cause​ male-patterned hair growth E. ​Seventy-eight-year-old female being treated for shingles

A. ​Fifty-year-old recently remarried male taking antihypertensive medication B. ​Thirty-four-year-old female with family history of blood clots taking an estrogen oral contraceptive C. ​Thirty-eight-year-old male recently started on an antidepressant D. ​Twenty-eight-year-old female started on acne medication known to cause​ male-patterned hair growth

A small nuclear weapon has been detonated in a nearby city. Which immediate symptoms of radiation sickness will the nurse assess in clients who have been​ exposed? A. ​Nausea, vomiting, and diarrhea. B. Anorexia and fatigue. C. Dilated pupils and aggression. D. Weight loss and fatigue.

A. ​Nausea, vomiting, and diarrhea.

A client was exposed to ionizing radiation. The nurse plans to administer which of the following​ medications? A. ​Potassium-iodine tablets. B. Salt tablets. C. Antibiotics. D. Calcium tablets.

A. ​Potassium-iodine tablets.

A charge nurse is reviewing the types of prescriptions with a newly licensed nurse. Which of the following prescriptions should the nurse include as an example of a standing prescription? Oxycodone 5 mg by mouth every 4 hr as needed for pain Furosemide 20 mg IV stat Acetaminophen 650 mg by mouth every 6 hr for temperature greater than 38.4° C (101.2° F) Diazepam 10 mg IV 30 min prior to procedure

Acetaminophen 650 mg by mouth every 6 hr for temperature greater than 38.4° C (101.2° F)

A nurse is caring for a client who reports severe back pain at 1400. The client's prescriptions include oxycodone extended - release 20 mg PO every 12 hr (last dose received at 600) and oxycodone immediate - release 5 mg PO every 4 hr PRN (last dose received at 2300 the day before). Which of the following actions should the nurse take? Contact the provider to request an order for a different pain medication. Administer oxycodone immediate-release 5 mg PO at 1600. Administer oxycodone immediate-release 5 mg PO now. Contact the provider to request an increase in the oxycodone extended-release dose.

Administer oxycodone immediate-release 5 mg PO now.

A nurse is preparing to administer a medication to a newly admitted client. The nurse should identify which of the following actions as a part of the assessment phase of the nursing process? Asking the client about a history of medication allergies Instructing the client about the medication's adverse effects Determining whether the medication should be administered with or without meals Monitoring the client's response to the medication

Asking the client about a history of medication allergies

A nurse is preparing to administer medications to a​ school-age child. What would the nurse​ do? A. Administer drugs while holding the child down. B. Allow the child to make decisions regarding how medications are taken. C. Provide a brief explanation on why the medication is important. D. Provide a lengthy explanation followed by quick drug administration.

B. Allow the child to make decisions regarding how medications are taken.

The nurse is preparing for medication administration to a group of clients. What is the best overall outcome for the​ clients? A. Clients will take the medications after receiving medication instruction. B. Clients will receive the best therapeutic outcome from the medications. C. Clients will state the reason they are receiving the medications. D. Clients will experience minimal side effects after taking the medications.

B. Clients will receive the best therapeutic outcome from the medications.

A client is a vegan vegetarian and does not eat any animal products. The healthcare provider has prescribed a medication contained in a gelatin capsule. The nurse understands that a gelatin capsule is made from animal products. Which nursing actions are​ indicated? Select all that apply. A. Do not tell the client the capsule contains gelatin. B. Collaborate with the prescriber to find an alternative medication. C. Check to see if the capsule can be opened for administration. D. Work to convince the client that this application does not violate vegetarian beliefs. E. Tell the client that the gelatin is an inactive ingredient in the medication.

B. Collaborate with the prescriber to find an alternative medication. C. Check to see if the capsule can be opened for administration.

A client with hypertension is receiving medication to lower his blood pressure. Which nursing action demonstrates the evaluation process related to medication​ administration? A. Asking the client if they have adhered to the prescribed treatment B. Determining that goals were not met 3 days following medication administration C. Administration of IV antihypertensive agents D. Determination of the​ client's baseline blood pressure

B. Determining that goals were not met 3 days following medication administration

The Joint Commission documented that client education was deficient on several​ medical-surgical units of a local hospital. A nursing committee was formed to address this problem. What is the best intervention to improve client​ education? A. Providing educational pamphlets about medications to the clients. B. Discussing medications each time they are administered to clients. C. Asking the healthcare providers to provide medication education to the clients. D. Requesting more frequent pharmacy consults for the clients.

B. Discussing medications each time they are administered to clients.

A nurse is reviewing medication records of older adults living in extended care. Which concepts about the pharmacotherapy of older adults should be​ considered? Select all that apply. A. ​Plasma-binding of drugs is greater in those over age 65. B. Drug distribution is slowed in the older adult. C. The​ half-life of many drugs is lessened in older adults. D. ​First-pass metabolism is reduced in the older adult. E. Changes in the kidney may result in decreased drug excretion.

B. Drug distribution is slowed in the older adult. D. ​First-pass metabolism is reduced in the older adult. E. Changes in the kidney may result in decreased drug excretion.

The nursing instructor teaches student nurses about the Food and Drug Administration​ (FDA) Pregnancy Categories. What should the nurse​ include? Select all that apply. A. Food and Drug Administration​ (FDA) Pregnancy Category C is safe to use during pregnancy B. Food and Drug Administration​ (FDA) Pregnancy categories provide a framework for safe use of drugs in pregnant women. C. Food and Drug Administration​ (FDA) Pregnancy categories for individual drugs seldom change once they are established. D. Food and Drug Administration​ (FDA) Pregnancy categories are based on studies using clinical human research trials. E. Food and Drug Administration​ (FDA) Pregnancy Category X has been associated with teratogenic effects.

B. Food and Drug Administration​ (FDA) Pregnancy categories provide a framework for safe use of drugs in pregnant women. E. Food and Drug Administration​ (FDA) Pregnancy Category X has been associated with teratogenic effects.

The nurse plans to teach a safety class to parents of toddlers about household exposure to medications. What should the nurse​ include? Select all that apply. A. Keep the toddler awake and observe for side effects of the medication. B. Keep all medications locked up and stored out of reach of the toddler. C. Use syrup of ipecac immediately if the toddler has ingested medication. D. Call the Poison Control Center for guidance with any medication ingested. E. Teach the toddler that medications are for adult use only.

B. Keep all medications locked up and stored out of reach of the toddler. D. Call the Poison Control Center for guidance with any medication ingested.

A nurse is planning a discussion of emergency preparedness with newly hired nurses. Which events would this nurse use as historical examples of​ terrorism? Select all that apply. A. The death of a client in a Texas hospital was confirmed to be due to the Ebola virus. B. Many people died as a result of sarin gas exposure in a Tokyo subway. C. ​Twenty-two confirmed or suspected cases of anthrax infection occurred as a result of the bacillus being sent through U.S. mail. D. There is typically a marked increase in overdoses during winter holidays. E. More people are contracting H1N1 avian influenza.

B. Many people died as a result of sarin gas exposure in a Tokyo subway. C. ​Twenty-two confirmed or suspected cases of anthrax infection occurred as a result of the bacillus being sent through U.S. mail.

Which nursing intervention would take priority following administration of a new​ medication? A. Measuring​ client's weight daily B. Monitoring the​ client's respiratory status C. Administering additional medications if side effects occur D. Evaluate the results of recent labs

B. Monitoring the​ client's respiratory status

Which statement about the nursing process is​ accurate? A. After selecting the nursing​ diagnosis, interventions are completed. B. Obtaining the outcomes is essential for goal attainment. C. ​Generally, goals are more measurable than outcomes. D. Goals involve very specific criteria that evaluate interventions.

B. Obtaining the outcomes is essential for goal attainment.

The nurse will administer medication to a​ school-age child. What is the best action by the​ nurse? A. Tell the child he will not be allowed to go to recess if the medication is not taken. B. Offer the child a choice of beverage with which to take the medication. C. Offer to play with the child prior to medication administration. D. Teach the child the action and expected side effects of the medication.

B. Offer the child a choice of beverage with which to take the medication.

The client uses Ginkgo biloba to enhance memory functioning but has not told the nurse about this herb. The healthcare provider orders warfarin​ (Coumadin) for this client. Which findings may the nurse find upon​ assessment? A. ​Headache, dizziness,​ sweating, and agitation. B. Petechiae and bleeding from the gums. C. An international normalized ratio​ (INR) of 3.0. D. A blood glucose of 56.

B. Petechiae and bleeding from the gums.

A​ client's genetic testing reveals a change in CYP2A6. The nurse would plan which interventions based on this​ discovery? Select all that apply. A. Teaching the client to avoid​ caffeine-containing drugs. B. Planning additional time and interventions to help the client stop smoking. C. Use of an alternative to the drug warfarin. D. Increased surveillance for the development of lung cancer. E. Frequent monitoring of blood pressure.

B. Planning additional time and interventions to help the client stop smoking. D. Increased surveillance for the development of lung cancer.

The client was exposed to cutaneous anthrax 2 weeks ago. What will the nurse see when assessing the​ client's skin? A. ​Small, fluid-filled​ vesicles, and​ later, small skin erosions B. Small skin lesions and​ later, black scabs C. Ulcerated areas and​ later, keloids D. Large pustules and​ later, reddish scabs

B. Small skin lesions and​ later, black scabs

A client has been admitted to the emergency department for treatment of poison ingestion. Gastric lavage and placement of activated charcoal has been prescribed. The nurse would withhold this treatment and collaborate with the prescriber if which instances​ occur? Select all that apply. A. The client vomits. B. The client becomes unconscious. C. The client discloses that lithium was the drug taken. D. Paramedics report that it has been 2 hours since the drug was taken. E. The client begins to pass copious amounts of watery diarrhea.

B. The client becomes unconscious. C. The client discloses that lithium was the drug taken. D. Paramedics report that it has been 2 hours since the drug was taken.

The nurse is assessing a newly admitted​ client's current medication. Which of the following is an example of objective​ data? A. The client lists the medications that have been prescribed. B. The nurse checks the prescription bottles the client has brought to the hospital. C. The nurse asks the healthcare provider what medications the client was currently taking. D. The​ client's wife tells the nurse what medications the client has been receiving.

B. The nurse checks the prescription bottles the client has brought to the hospital.

An American Indian client has been admitted to the hospital for chemotherapy. At any given​ time, five family members are in the​ client's room. The nurse tells the client that according to hospital​ policy, only two visitors at a time are allowed. What does the best analysis by the nurse manager reveal about the​ nurse's action? A. The nurse should have allowed the client to have as many family members as she wanted to be present. B. The nurse should have assessed the​ client's preferences about how many family members she wanted to be present. C. The nurse should have called the healthcare provider and obtained an order for additional family members to be present. D. This was the correct​ action; the nurse was following protocol by informing the client about hospital policy.

B. The nurse should have assessed the​ client's preferences about how many family members she wanted to be present.

A nurse is planning to teach a client about the relationship of complementary and alternative therapies to pharmacology What should the nurse​ include? A. They serve as competitors and should not be promoted. B. They can reduce client medication needs. C. They are more effective than medication use. D. They have​ little-to-no value in disease prevention and treatment.

B. They can reduce client medication needs.

The nurse is managing care for several clients at a diabetic treatment center. The nurse understands that which of the following is the priority nursing​ intervention? A. To include any cultural or ethnic preferences in the administration of the medication B. To return the client to an optimum level of wellness while limiting adverse effects related to the​ client's medical diagnosis C. To administer the correct medicine to the correct client at the correct dose and the correct time via the correct route D. To answer any questions the client may have about the​ medicine, or any possible side effect of the medication

B. To return the client to an optimum level of wellness while limiting adverse effects related to the​ client's medical diagnosis

Medication reconciliation has been started for a newly admitted client. At which points would the nurses and others caring for this client check this​ list? Select all that apply. A. Each time that medications are administered to the client. B. When initial admission orders are received. C. When the client is transferred to a different unit within the hospital. D. When the client is discharged. E. If a medication error occurs.

B. When initial admission orders are received. C. When the client is transferred to a different unit within the hospital. D. When the client is discharged.

A nursing student asks a nurse how gender influences pharmacology. How should the nurse​ respond? A. Studies indicate that men and women suffer from​ Alzheimer's disease in equal numbers. B. Women tend to seek medical care earlier than men do. C. Heart disease has traditionally been thought of as a​ woman's disease. D. Since the​ 1980s, the FDA has mandated that research studies include both male and female subjects.

B. Women tend to seek medical care earlier than men do.

The nursing instructor is teaching student nurses about the use of viruses in a bioterrorism attack. The nurse determines that learning has occurred when the students make which​ statement? A. ​"The Centers for Disease Control and Prevention​ (CDC) has a plan to vaccinate Americans against most​ viruses." B. ​"A bioterrorist attack with viruses is a real threat to​ Americans." C. ​"Most Americans have already been vaccinated against the lethal​ viruses." D. ​"Actually, a bigger concern is a nuclear weapon exploding in a​ city."

B. ​"A bioterrorist attack with viruses is a real threat to​ Americans."

The nurse teaches a class to clients about how to help prevent medication errors when in the hospital. What is the most important question for the nurse to ask the​ clients? A. ​"Do you have a friend to verify that you are receiving the correct​ medication?" B. ​"Do you know the names of all the medications you​ take?" C. ​"Do you trust your healthcare provider to order the correct​ medication?" D. ​"Do you know what your illness​ is, and if you will need​ surgery?"

B. ​"Do you know the names of all the medications you​ take?"

The older adult asks the nurse how dietary supplements will help support health. What is the best response by the​ nurse? A. ​"Soy isoflavone will help prevent​ Alzheimer's disease." B. ​"Fish oil will help to enhance your brain​ function." C. ​"Dietary supplements will help support and maintain​ hydration." D. ​"Chromium will help you achieve and maintain optimum​ weight."

B. ​"Fish oil will help to enhance your brain​ function."

The nurse manager would determine that a nurse understands culturally sensitive care if which statements are​ made? Select all that apply. A. ​"Since all of our clients have the same​ illnesses, I sometimes get​ bored." B. ​"I have been helping my client understand how to choose foods from our diet​ menu." C. ​"I am concerned because my client needs sterile dressing changes but has no running water at​ home." D. ​"My client and I worked out a way to get her medications delivered to her at​ home." E. ​"I invited a group from my church to come and sing hymns for our​ clients."

B. ​"I have been helping my client understand how to choose foods from our diet​ menu." C. ​"I am concerned because my client needs sterile dressing changes but has no running water at​ home." D. ​"My client and I worked out a way to get her medications delivered to her at​ home."

The client comes to the emergency department with an anxiety attack. He or she tells the nurse he heard that there was another anthrax attack in the capitol and is concerned about running out of medications. What is the best response by the​ nurse? A. ​"Your health is in danger due to the​ anxiety; we really need to focus on reducing your anxiety​ now." B. ​"The Centers for Disease Control and Prevention​ (CDC) maintains a large stockpile of medications for us in case that​ occurs." C. ​"You don't need to worry about another attack at​ all; I think our government can take care of​ us." D. ​"I'm sure the Centers for Disease Control and Prevention​ (CDC) has contingency plans in the event of an anthrax​ attack."

B. ​"The Centers for Disease Control and Prevention​ (CDC) maintains a large stockpile of medications for us in case that​ occurs."

The nurse prepares to teach clients about the safety and efficacy of herbal preparations. Which statement would be included in the best plan by the​ nurse? A. ​"The label on the herbal preparation is required by the Dietary Supplement Health and Education Act​ (DSHEA) to be​ accurate." B. ​"The manufacturer does not have to prove the safety or efficacy of the herbal​ preparation." C. ​"Herbal preparations have to meet the same safety and efficacy standards as prescription and​ over-the-counter (OTC)​ drugs." D. ​"Herbal preparations must be tested for safety and efficacy prior to​ marketing."

B. ​"The manufacturer does not have to prove the safety or efficacy of the herbal​ preparation."

The nurse is teaching a support group for caretakers of older adult clients. The focus is medication compliance. The nurse determines that learning has occurred when the caregivers make which​ response? A. ​"We should give them more education about the medicine so they will take​ it." B. ​"We should use a medication management box so they​ won't forget to take​ it." C. ​"We should ask the doctor if all the medication is really​ necessary." D. ​"We should crush their medicine and put it in applesauce so they will swallow​ it."

B. ​"We should use a medication management box so they​ won't forget to take​ it."

The nurse was very busy and unfamiliar with a new​ medication, but administered it anyway. Later the nurse looked up the medication. How does the nurse manager evaluate this​ behavior? A. This was acceptable as long as the nurse looked up the action and side effects of the drug later. B. The nurse manager was partially at fault because the nursing unit was understaffed and the nurse was too busy. C. An error could have occurred because the nurse was unfamiliar with the medication. D. An error did occur because the nurse could have administered the medication via the incorrect route.

C. An error could have occurred because the nurse was unfamiliar with the medication.

The nursing supervisor tells a nurse that the medication error the nurse made yesterday has been determined to be a sentinel event. What should the nurse expect to​ occur? Select all that apply. A. Her employment will be terminated. B. Her personal malpractice insurance company will be notified. C. An immediate investigation will occur. D. Interventions to prevent the error from occurring again will quickly be put in place. E. A​ root-cause analysis will be performed.

C. An immediate investigation will occur. D. Interventions to prevent the error from occurring again will quickly be put in place. E. A​ root-cause analysis will be performed.

The nurse has several educational pamphlets for the client about medications the client is receiving. Prior to giving the client these​ pamphlets, what is the most important assessment by the​ nurse? A. Assess the​ client's ability to pay for the medication. B. Assess the​ client's religious attitudes toward medicine. C. Assess the​ client's reading level. D. Assess the​ client's cultural bias toward taking medicine.

C. Assess the​ client's reading level.

A nurse is teaching a student nurse about the active ingredients of herbal medicines. Which of the following indicates that teaching was​ effective? A. Are found only in the stems and leaves. B. Are found only in the root system. C. Can be found in all parts of the herb. D. Are found only in the rhizome.

C. Can be found in all parts of the herb.

A nursing instructor is teaching nursing students about the FDA Pregnancy Drug Categories. The instructor asks the students to what category a drug for which research has shown an adverse effect in animals but not in pregnant women would be categorized as. The instructor knows learning has occurred with which student​ response? A. Category D B. Category C C. Category B D. Category A

C. Category B

The nurse assesses an adverse effect of a medication that has been administered. Who should the nurse report this adverse effect​ to? A. Food and Drug​ Administration's (FDA) Safe Medicine Website. B. Food and Drug​ Administration's (FDA) Med MARX Website. C. Food and Drug​ Administration's (FDA) MedWatch Website. D. Food and Drug​ Administration's (FDA) Adverse Event Website.

C. Food and Drug​ Administration's (FDA) MedWatch Website.

A nurse is planning to teach a client about a new medication. What is the best teaching​ method? A. Instruct the client that their local pharmacy will teach them about this medication. B. Provide oral drug information and instructions as opposed to written. C. Give the client oral and written drug information and instructions. D. Leave written drug information and instructions at the bedside.

C. Give the client oral and written drug information and instructions.

A nurse has admitted a new client to the unit. Which concepts should the nurse use when developing a nursing​ diagnosis? Select all that apply. A. Base the nursing diagnosis on the medical diagnosis B. Focus on what the nurse needs to help the client return to health C. Include the client in the identification of needs D. Consider the​ client's response to the current health problem E. Be certain the diagnosis is measureable

C. Include the client in the identification of needs D. Consider the​ client's response to the current health problem

A nursing instructor is planning to discuss the Dietary Supplement Health and Education Act​ (DSHEA) of 1994 with a group of nursing students. The instructor plans to include which of the​ following? A. It requires that herbal products undergo the same rigorous testing as drugs do under the FDA. B. It mandates that herbal product labels contain accurate information. C. It mandates that herbal product labels state that the products are not intended to​ diagnose, treat,​ cure, or prevent disease. D. It ensures that herbal products provide proof of their intended effects.

C. It mandates that herbal product labels state that the products are not intended to​ diagnose, treat,​ cure, or prevent disease.

The nurse is administering medications to an older adult. Which laboratory tests are most important for the nurse to assess prior to the administration of​ medication? A. Arterial blood gases​ (ABGs) and basic metabolic panel B. Lipid panel and thyroid function tests C. Kidney and liver function tests D. Complete blood count​ (CBC) and electrolytes

C. Kidney and liver function tests

The clinic nurse will immediately alert the healthcare provider when which category X drugs are identified on the medication record of a recently diagnosed pregnant​ patient? Select all that apply. A. Tetracycline B. ACE inhibitor antihypertensive medication C. Methotrexate D. Isotretinoin​ (Accutane) E. Oral contraceptives

C. Methotrexate D. Isotretinoin​ (Accutane) E. Oral contraceptives

The nurse provides care for elderly clients in an assisted living facility. What does the nurse assess as a primary contributing factor for drug toxicity in the​ elderly? A. Older adults often abuse alcohol. B. Older adults are frequently constipated. C. Older adults are frequently dehydrated. D. Older adults have decreased stomach acid.

C. Older adults are frequently dehydrated.

The nurse administers potassium iodide​ (KI) tablets to a client who has been exposed to radiation from a nuclear weapon. What is the rationale for administering potassium iodide​ (KI) to this​ client? A. Prevent liver cancer B. Prevent renal cancer C. Prevent thyroid cancer D. Prevent brain cancer

C. Prevent thyroid cancer

A Hispanic man has been diagnosed with​ attention-deficit/hyperactivity disorder​ (ADHD), and is taking methylphenidate​ (Ritalin). Even though the drug helps with focus and​ grades, the client will not go to the school office at noon for his medication. Which statement best describes the result of the​ nurse's evaluation? A. The adolescent really does not need an additional dose of methylphenidate​ (Ritalin) at school. B. The adolescent has developed alternative coping mechanisms to increase his focus during classes. C. The adolescent is embarrassed about having to take medicine at​ school; it is a social stigma. D. The adolescent is fearful that this drug may be a​ "gateway drug" and he will abuse other substances.

C. The adolescent is embarrassed about having to take medicine at​ school; it is a social stigma.

The healthcare provider has prescribed quetiapine​ (Seroquel) for the client with chronic auditory hallucinations. The client has stopped taking the medication. The nurse incorrectly uses the diagnosis of​ "noncompliance." In which situation would this diagnosis be​ appropriate? A. The client did not understand why the medication was prescribed. B. The client was unsure about how to order a refill for the prescription. C. The client made an informed decision not to take the medication. D. The client reported a physical change as the reason for stopping the medication.

C. The client made an informed decision not to take the medication.

A nurse is developing a care plan for a client. Which client outcome statements are correctly​ formatted? Select all that apply. A. The client will understand the effects of the medication administered prior to discharge. B. The nurse will administer all medications with ten minutes of their scheduled time. C. The client will identify two adverse effects of enoxaprarin​ (Lovenox) prior to​ self-administering the drug. D. The client will verbalize the storage requirements for NPH insulin prior to discharge. E. The healthcare provider will discuss the desired effects of discharge medications with the client the evening before discharge.

C. The client will identify two adverse effects of enoxaprarin​ (Lovenox) prior to​ self-administering the drug. D. The client will verbalize the storage requirements for NPH insulin prior to discharge.

All of the clients listed have cancer and are receiving chemotherapy. Which client does the nurse evaluate as having the highest probability for a​ remission? A. The client who is also seeing a psychiatrist for treatment of depression B. The client who is a former healthcare provider C. The client with a support group of cancer survivors D. The wealthy client who can afford the best medical care available

C. The client with a support group of cancer survivors

The nurse in the emergency department administers an adult dose of an antibiotic to a​ 3-month-old baby. As a​ result, the baby suffers permanent brain damage. What best describes the effect of this error on the healthcare​ facility? A. The reputation of the healthcare facility will suffer. B. The professional license of the nurse will be lost. C. The morale of the staff involved will be decreased. D. The healthcare facility will pay a very large settlement.

C. The morale of the staff involved will be decreased.

The nurse plans to administer medication to a preschool child. Which approach indicates the nurse has an understanding of growth and​ development? A. The child does better with verbal instruction than with play instruction. B. There should be no need to restrain a child of this age. C. Use a brief​ rationale, followed by quick administration of the medication. D. The child is often more cooperative if the parent is not in the room.

C. Use a brief​ rationale, followed by quick administration of the medication.

The client is receiving a​ beta-blocker medication. The nurse has done medication education and provided the client with printed information to take home. During the next​ appointment, the nurse notes that the client is not taking the medication properly. What is a therapeutic assessment question to ask this​ client? A. ​"Don't you understand how important it is to take the​ medicine?" B. ​"Why didn't you take your medicine as we talked​ about?" C. ​"Are you able to read and comprehend the printed​ information?" D. ​"Do I have to inform your healthcare provider about your​ noncompliance?"

C. ​"Are you able to read and comprehend the printed​ information?"

The nurse makes a medication error and a client dies. In​ court, the attorney for the family of the deceased client asks the nurse if she followed standards of care in administering the medication. How would the attorney phrase this​ question? A. ​"Did you follow agency guidelines as in previous​ circumstances?" B. ​"Did you follow the healthcare​ provider's orders and​ double-check them before​ administration?" C. ​"Did you do what another nurse would have done under similar​ circumstances?" D. ​"Did you do the three checks and follow the five rights as taught in​ school?"

C. ​"Did you do what another nurse would have done under similar​ circumstances?"

The nurse is preparing an educational plan for parents about how to protect their children if a bioterrorist attack occurs. What is the best information to​ include? A. ​"Realistically, there is nothing that can be​ done." B. ​"Plan to call the Centers for Disease Control and Prevention​ (CDC) if an attack​ occurs." C. ​"Follow the Centers for Disease Control and Prevention​ (CDC) guidelines for​ immunizations." D. ​"Don't worry, the Centers for Disease Control and Prevention​ (CDC) has everything under​ control."

C. ​"Follow the Centers for Disease Control and Prevention​ (CDC) guidelines for​ immunizations."

The nurse has been teaching a client about herbal preparations and determines that additional teaching is required when the client makes which​ statement? A. ​"I need to be careful about where I store my herbal​ product." B. ​"I should check with you before using an herbal​ product." C. ​"Herbal products usually contain only one active​ ingredient." D. ​"Herbal preparations are available in solid and liquid​ forms."

C. ​"Herbal products usually contain only one active​ ingredient."

The client is receiving an oral antibiotic as treatment for cellulitis of the lower extremity. The​ client's outcome is​ "Client will state a key point about antibiotic treatment for​ cellulitis." Which statement would the nurse evaluate as best indicating this outcome has been​ met? A. ​"If the swelling​ continues, I can apply an ice​ pack." B. ​"If the pain gets too​ bad, I can take my prescribed pain​ medication." C. ​"I need to take all the pills even if my leg looks​ better." D. ​"I must keep my leg elevated until the swelling goes​ down."

C. ​"I need to take all the pills even if my leg looks​ better."

The pregnant client plans to breastfeed her baby. She asks the nurse about the use of herbal products during breastfeeding. What is the best response by the​ nurse? A. ​"Herbal products are considered​ natural, so it should be fine to use them during​ breastfeeding." B. ​"This should be​ fine, as long as there is at least 12 hours between the time you use the product and when you​ breastfeed." C. ​"Most drugs can be transferred to the infant during​ breastfeeding, so this is not​ recommended." D. ​"Be sure to check the label to see if the herbal product could be used during​ breastfeeding."

C. ​"Most drugs can be transferred to the infant during​ breastfeeding, so this is not​ recommended."

A pregnant client asks the nurse about changing from her prescription antidepressant medication to St.​ John's wort because it is natural. What is the best response by the​ nurse? A. ​"Yes, you can​ change, but let your doctor know at your next​ appointment." B. ​"No, herbal preparations are just not safe to take during​ pregnancy." C. ​"St. John's wort is a​ drug, and this should be discussed with your healthcare​ provider." D. ​"It should be okay because your baby has been exposed to an​ antidepressant."

C. ​"St. John's wort is a​ drug, and this should be discussed with your healthcare​ provider."

The nurse teaches a class about medication used during pregnancy to pregnant women. The nurse determines that additional instruction is required when a class participant makes which​ response? A. ​"Exposure to teratogens can result in my​ baby's death or in​ malformations." B. ​"It is important to not take​ over-the-counter (OTC) drugs during my​ pregnancy." C. ​"The baby can only be harmed by medications during the first​ trimester." D. ​"If I breastfeed my​ baby, drugs can come through my breast​ milk."

C. ​"The baby can only be harmed by medications during the first​ trimester."

The client tells the nurse that she is concerned about terrorist activity and questions if everyone should be immunized against smallpox. What is the best response by the​ nurse? A. ​"I really do not think our country has enough vaccine to do​ this." B. ​"The vaccine has some serious side​ effects, but this is probably a good​ idea." C. ​"The vaccine has side​ effects, which are serious and could kill many​ people." D. ​"Don't be so​ concerned; if an attack​ comes, we will immunize people​ then."

C. ​"The vaccine has side​ effects, which are serious and could kill many​ people."

The client confides in the nurse that he or she is drawn to complementary and alternative medicine because it promotes the​ "whole person." The client would like spirituality and prayer included in his or her plan of care. What is the best response by the​ nurse? A. ​"I think your spiritual concerns are best left up to you and your​ minister." B. ​"What exactly do you mean by spirituality and​ prayer?" C. ​"We will include spirituality in your plan. Would you like to say a​ prayer?" D. ​"We usually do not pray with​ clients; is there something you are worried​ about?"

C. ​"We will include spirituality in your plan. Would you like to say a​ prayer?"

The client tells the nurse that he or she has been taking herbal preparations to boost his or her immune system functioning. He or she does not know the names of the preparations. What is the best assessment question for the nurse to​ ask? A. ​"Would you please tell your doctor about the herbs during the next​ visit?" B. ​"Would you please ask your wife or husband to call the hospital pharmacist with the​ names?" C. ​"Would you please have your wife or husband bring the bottles to the​ hospital?" D. ​"Would you please ask your wife or husband to discuss this with me during her next​ visit?"

C. ​"Would you please have your wife or husband bring the bottles to the​ hospital?"

A student nurse asks a nursing instructor which federal agency responsible for reviewing all medication errors reports. What is the nursing​ instructor's best​ response? A. Centers for Disease Control​ (CDC) B. Medication errors are never acceptable. National Coordinating Council for Medication Error Reporting and Prevention​ (NCC MERP) C. ​FDA's Division of Medication Error Prevention and Analysis​ (DMEPA) D. Risk Management department at the healthcare facility in which it occurred

C. ​FDA's Division of Medication Error Prevention and Analysis​ (DMEPA)

A nurse is administrating medications to four clients. The nurse should identify which of the following nursing actions as a part of the evaluation phase of the nursing process? Collecting information about a client's pain level following administration of a narcotic Taking the blood pressure of a client before administering an antihypertensive medication Lowering the level of a client's bed before administering a benzodiazepine medication Instructing a client to rinse their mouth following administration of an inhalation corticosteroid

Collecting information about a client's pain level following administration of a narcotic

A nurse working in a medical - surgical unit is preparing to administer medications to a client. The nurse plans to use two forms of identification to verify that she has the right client. Which of the following actions can the nurse take to identify the client? Select all that apply. Compare the name on the client's wristband with the name in the medication administration record (MAR). Ask the client to state his date of birth. Check the room number in the medication administration record (MAR) with the room number of the client. Ask the client to state his name. Use the bar code scan to identify the client.

Compare the name on the client's wristband with the name in the medication administration record (MAR). Ask the client to state his date of birth. Ask the client to state his name. Use the bar code scan to identify the client.

A student nurse asks a nurse what items are found within the Strategic National Stockpile​ (SNS). What is the​ nurse's best​ response? A. ​Two-way radios B. Prepared meals C. Drinking water D. Antibiotics

D. Antibiotics

The nurse is reviewing the steps of the nursing process with a student. The nurse is aware that it is most important to be accurate in which portion of the nursing​ process? A. Evaluation B. Diagnosis C. Planning D. Assessment

D. Assessment

The healthcare provider has prescribed a nitroglycerine​ (Nitrodur) patch for the client. The nurse understands that which of the following is the best outcome for this client as it relates to use of the​ medication? A. Client will be able to identify the expiration date of the medication prior to discharge. B. Client will verbalize three side effects of the medication prior to discharge. C. Client will state the reason for receiving the medication prior to discharge. D. Client will demonstrate correct application of the patch prior to discharge.

D. Client will demonstrate correct application of the patch prior to discharge.

The nurse is teaching a class on anthrax to a group of emergency response workers. The nurse plans to include which of the​ following? A. Anthrax most commonly affects wild rodents such as​ mice, rats,​ squirrels, and chipmunks. B. Cutaneous anthrax is serious because it quickly spreads by​ person-to-person contact. C. Anthrax is a deadly​ bacterium; the most common and deadly form is gastrointestinal anthrax. D. Cutaneous anthrax is the most common​ form, but inhaled anthrax is the most lethal form.

D. Cutaneous anthrax is the most common​ form, but inhaled anthrax is the most lethal form.

The client takes St.​ John's wort for depression but does not tell the healthcare provider. The provider prescribes an SSRI. The nurse should monitor for which of the​ following? A. Serum potassium of 6.0 B. Sedation C. Serum glucose level of 340 D. Dizziness

D. Dizziness

A nursing student asks a nurse how pharmacotherapy in the older adult is different than a​ middle-aged adult. How would the nurse​ respond? A. ​Generally, drug doses should be increased due to prolonged drug metabolism. B. Increased body water can lead to a higher risk of drug toxicity. C. Plasma levels are​ increased, leading to a heightened drug response. D. Drug absorption is slower due to increased gastric pH.

D. Drug absorption is slower due to increased gastric pH.

The nurse teaching a​ pre-conception class would tell participants that they should be most careful about exposure to drugs during which stage of​ pregnancy? A. Third trimester B. Preimplantation period C. Each stage is equal in risk D. Embryonic

D. Embryonic

A nurse is reviewing safe medication administration with a student nurse. What should the nurse plan to include in the​ teaching? A. Administer medications intramuscularly when a client refuses to take it orally B. Discontinue a medication at the request of a client C. Use abbreviations while charting to save time D. Give medications within the time frame specified by hospital policy

D. Give medications within the time frame specified by hospital policy

A nurse manager is discussing medication errors with a group of nurses. Which statement by the nurses indicates the teaching was​ effective? A. A nurse who observes the five rights will prevent all medication errors from occurring. B. Nurses are always liable when a medication error occurs. C. An incorrect dose​ (based on​ weight) is​ ordered, dispensed, and administered to a client. The administering nurse and ordering clinician would be the only parties held accountable. D. Handwritten orders are more frequently associated with medication errors than are typed orders.

D. Handwritten orders are more frequently associated with medication errors than are typed orders.

The client is receiving albuterol​ (Proventil) for treatment of bronchospasm related to asthma. What is the primary nursing intervention as it relates to this​ medication? A. Provide the client with​ age-appropriate education about albuterol​ (Proventil). B. Monitor the client for nausea and headache. C. Monitor the​ client's serum drug levels. D. Monitor the client for relief of bronchospasms.

D. Monitor the client for relief of bronchospasms.

The client has been exposed to a nerve agent. For which symptoms will the nurse most likely​ assess? A. Dilated pupils and increased heart rate. B. Pinpoint pupils and decreased blood pressure. C. Rapid Tachypnea and​ cold, clammy skin. D. Salivation and involuntary urination.

D. Salivation and involuntary urination.

The nurse is preparing medications for a group of clients. Another nurse begins telling the nurse about her recent engagement. What is the best action by the first​ nurse? A. Stop preparing medications until the second nurse has finished talking about her engagement. B. Ask the second nurse to help with administering medications so they can have more time to talk. C. Continue to prepare the medications for administration and pretend to listen to the first nurse. D. Tell the second nurse that the conversation is distracting and she must stop talking while medications are being prepared.

D. Tell the second nurse that the conversation is distracting and she must stop talking while medications are being prepared.

A nurse is preparing to administer medications to a pregnant client. Which classification of drugs would the nurse refuse to​ administer? A. Cautionary B. Category A C. Category B D. Teratogenic

D. Teratogenic

A nurse is assessing a client recently admitted to the unit. The nurse understands that which assessment identifies the collection of objective​ data? A. The client states he or she is anxious. B. The client informs the nurse that he or she weighs 150 pounds. C. The client rates his or her pain a 5 on a 0-10 pain scale. D. The client has a wound measured at 5 cm in length.

D. The client has a wound measured at 5 cm in length.

The client is being treated for a cardiac dysrhythmia with amiodarone​ (Cordarone). The client has elevated liver function tests​ (LFTs). Which assessment finding would the nurse suspect as contributing to the elevated​ LFTs? A. The client was taking​ Kava-Kava for anxiety. B. The client was taking St.​ John's wort for depression. C. The client was taking gingko biloba for memory problems. D. The client was taking Echinacea to treat cold symptoms.

D. The client was taking Echinacea to treat cold symptoms.

A nurse is preparing to administer a new drug that was just prescribed by the healthcare provider. The nurse recognizes that the wrong concentration of the drug was sent by pharmacy. What is the​ nurse's best​ response? A. The nurse does not report or document the​ error, since the error did not result in any harm to the patient. B. The nurse does not report the​ error, because the error was caught and corrected prior to drug administration. C. The nurse reports the error to the healthcare provider and the charge nurse but does not document the error due to possible legal action. D. The nurse informs the​ client, documents the error as per hospital​ policy, and notifies the healthcare provider.

D. The nurse informs the​ client, documents the error as per hospital​ policy, and notifies the healthcare provider.

A nurse is administering medications to a group of clients. Which situation is an example of a medication​ error? A. A medication is administered to a client with no​ allergies, yet an anaphylactic response occurs. B. A medication is administered in liquid form instead of tablet form due to the​ client's difficulty swallowing. C. A client experiences unexpected hypotension as a result of medication administration. D. The wrong dose of a medication is drawn up but is caught and corrected prior to administration.

D. The wrong dose of a medication is drawn up but is caught and corrected prior to administration.

A client asks a nurse why herbal remedies are popular now. What is the nurse best​ response? A. Most insurance policies cover them. B. They are more effective for treating bacterial infections. C. Their popularity is​ decreasing, not increasing. D. They are being marketed very aggressively.

D. They are being marketed very aggressively.

The nurse commits a medication error. The nurse documents the error in the​ client's record and completes the incident report. What does the nurse recognize as the primary reason for doing​ this? A. To protect the client from further harm B. To protect the healthcare facility from litigation C. To protect the nurse from liability D. To verify that the​ client's safety was protected

D. To verify that the​ client's safety was protected

The nurse is on a committee to reduce medication errors in a large healthcare facility. What is a recommendation the nurse proposes that will most likely help to reduce medication​ errors? A. Use robots to prepare all medications for administration by the nurse. B. Designate nurses whose only function is to administer medication. C. Train medication technicians to administer medications. D. Use​ automated, computerized cabinets on all nursing units.

D. Use​ automated, computerized cabinets on all nursing units.

A nurse is administering medications to a client. The client​ states, "I've never taken that yellow pill​ before". What should the nurse do​ first? A. Contact the health provider to verify it is the correct medication and dose. B. Reassure the client that the nurse has triple checked the drug so it is safe to take. C. Tell the client that some are made by different pharmaceutical companies and may look different. D. Verify the order and​ double-check the label.

D. Verify the order and​ double-check the label.

A nursing instructor is teaching a nursing student about human DNA sequences. The student demonstrates understanding by making with of the following​ statements? A. ​"Asian Americans are the ethnic group known to be slow​ acetylators." B. ​"Due to enzyme​ polymorphism, Hispanics are less likely to metabolize codeine to​ morphine." C. ​"Only 2% of human DNA is different among the different​ ethnicities." D. ​"Even though human genetic differences are​ small, significant differences can be seen with drug​ metabolism."

D. ​"Even though human genetic differences are​ small, significant differences can be seen with drug​ metabolism."

Clients at a senior citizen center have asked the nurse to do a presentation on herbal preparations. Which statement would be included in the best plan by the​ nurse? A. ​"Herbal preparations can be dangerous if you are allergic to​ them." B. ​"As long as the herbal preparation has been tested in the clinical​ setting, it is​ safe." C. ​"Herbal preparations are safe as long as you carefully read the​ label." D. ​"Herbal preparations actually are​ drugs; you must be careful with​ them."

D. ​"Herbal preparations actually are​ drugs; you must be careful with​ them."

The client tells the nurse that he or she plans to take St.​ John's wort to treat his or her depression. What is the best response by the​ nurse? Select all that apply. A. ​"That should be fine as long as you are not​ suicidal." B. ​"St. John's wort is successfully used in Europe for minor​ depression." C. ​"It would be a good idea to try this before paying for a prescription​ medication." D. ​"It would be better to have a psychiatric assessment​ first." E. ​"Herbal preparations can interact with many other​ medications."

D. ​"It would be better to have a psychiatric assessment​ first." E. ​"Herbal preparations can interact with many other​ medications."

The nurse is reviewing the steps of the nursing process with a student. The nurse knows that the student understands the teaching when the student correctly lists which of the following as the correct order of the nursing​ process? A. ​Assessment, planning, establish​ objectives, communication, evaluation B. Establish nursing​ diagnosis, assessment,​ intervene, collaborate, evaluation C. Establish​ goals, assessment,​ intervention, planning, communication D. ​Assessment, establish nursing​ diagnosis, planning,​ interventions, evaluation

D. ​Assessment, establish nursing​ diagnosis, planning,​ interventions, evaluation

Calcium carbonate (TUMS)

Drug class: Antacid MOA: neutralize gastric acidity uses: relief of acid indigestion, heartburn, sour stomach, and GI upset associated with these symptoms; calcium supplement

Conjugated estrogens/medroxyprogesterone acetate (Prempro)

Drug class: Estrogen and Progestin Combination MOA: Conjugated estrogen: development and maintenance of the female reproductive system and secondary sexual characteristics Postmenopausal → modulate the pituitary secretion of gonadotropins, LH, and FSH through a negative feedback system; estrogen replacement reduces elevated levels of these hormones in postmenopausal women. Progestrin (medroxyprogesterone) → transforms a proliferative endometrium into a secretory endometrium uses: Prevention of postmenopausal osteoporosis; Vulvar and vaginal atrophy & vasomotor symptoms associated with menopause (low estrogen)

Ethinyl estradiol/norgestimate (Ortho Tri-Cyclen)

Drug class: Estrogen and Progestin Combination Contraceptive MOA: inhibit ovulation via a negative feedback mechanism on the hypothalamus, prevents egg from fully developing Uses: acne and birth control

Ibuprofen (Advil/Motrin)

Drug class: NSAIDS MOA: Reversibly inhibits COX-1 and 2 enzymes uses: Inflammatory diseases and rheumatoid disorders, mild to moderate pain, fever, dysmenorrhea, osteoarthritis

Insulin aspart (Novolog)

Drug class: fast acting insulin MOA: bind to the insulin receptors on muscle and fat cells and lower blood glucose by facilitating the cellular uptake of glucose and simultaneously inhibiting the output of glucose from the liver. Uses: improve glycemic control of Diabetes mellitus, types 1 and 2

Albuterol (Proventil, Ventolin, Proair)

Drug class: inhaled beta 2 agonist MOA: works to relax the bronchial smooth muscle/ inhibits the release of immediate hypersensitivity mediators from cells indications: treatment/prevention of bronchospasm (acute or severe) in patients with reversible obstructive airway disease, including exercise-induced bronchospasm.

Insulin detemir (Levemir)

Drug class: insulin, long acting MOA: exert their specific action through binding to insulin receptors. Receptor-bound insulin lowers blood glucose by facilitating cellular uptake of glucose into skeletal muscle and fat and by inhibiting the output of glucose from the liver. uses: improve glycemic control of Diabetes mellitus, types 1 and 2

Glimepiride (Amaryl)

Drug class: sulfonylureas MOA: lowers blood sugar by causing the release of your body's natural insulin via the pancreas uses: Diabetes mellitus, type 2, adjunct to diet & exercise

A nurse should use which of the following units when measuring weight? Gram Liter Meter Milliliter

Gram

A nurse on a medical unit is assisting with the orientation of a newly licensed nurse. The nurse should remind the newly licensed nurse to have a second nurse review the dosage of which of the following medications prior to administration? Heparin Acetaminophen Acetylcysteine Hydroxychloroquine

Heparin

A nurse is preparing to administer insulin subcutaneously to a client. The nurse should document the administration of the medication immediately after which of the following actions? Taking the insulin from the automated dispensing machine Injecting the insulin Checking the client's blood glucose level Checking the correct dosage of the insulin

Injecting the insulin

A nurse is teaching a class about error - prone abbreviations in medication prescriptions. The nurse should include that which of the following prescriptions has acceptable abbreviations? Metoclopramide 10 mg IM Q6H PRN nausea Desipramine 100 mg PO QD Enoxaparin 30 mg sub q 2 hr before surgery MSO4 10 mg IM Q3H PRN pain

Metoclopramide 10 mg IM Q6H PRN nausea

A nurse is reviewing a client's prescriptions. The nurse should contact the provider to clarify which of the following prescriptions? Phenytoin 100 mg PO every 8 hr Morphine 2.5 mg IV bolus PRN for incisional pain Regular insulin 7 units subcutaneous 30 min before breakfast and dinner Lisinopril 20 mg PO every 12 hr. Hold for systolic BP less than 110 mm Hg

Morphine 2.5 mg IV bolus PRN for incisional pain

A nurse is caring for a client who received lisinopril 30 minutes ago and is now reporting dizziness and headache. Which of the following actions should the nurse take first? Obtain the client's vital signs. Notify the provider. Document the client's response in the medical record. Tell the client to change positions slowly.

Obtain the client's vital signs.

A nurse is assessing a client following administration of an antibiotic. The nurse should identify that which of the following findings is a manifestation of an anaphylactic reaction to the medication? Swollen lips Hypertension Low heart rate Constipation

Swollen lips

A nurse is preparing to administer digoxin 225 mcg for a pediatric client who has a heart rate above 90/min. Which of the following actions should the nurse take to ensure administration of the right dose? Select all that apply Validate that the dosage is within the safe range. Confirm the medication amount is appropriate for the child. Verify that the medication is not expired. Check the client's heart rate prior to administration. Document the administration in the medication administration record.

Validate that the dosage is within the safe range. Confirm the medication amount is appropriate for the child.

A nurse is preparing to administer a high - alert pain medication to a client. Which of the following actions should the nurse perform during the planning stage of medication administration? Assess the effectiveness of the pain medication. Verify the dosage calculation with another nurse. Teach the client about the action of the medication. Ask the client to state their name and birthdate.

Verify the dosage calculation with another nurse.

A charge nurse is teaching a newly licensed nurse about medication reconciliation. Which of the following information should the charge nurse include in the teaching? Perform medication reconciliation daily during a client's hospitalization. Only newly prescribed medications need to be reviewed during a medication reconciliation. Vitamins, supplements, and over-the-counter (OTC) medications should be included in a medication reconciliation. The goal of medication reconciliation is to minimize the financial impact of prescription medications to the client.

Vitamins, supplements, and over-the-counter (OTC) medications should be included in a medication reconciliation.

A nurse is preparing to administer medication to a client who has a prescription for decussate sodium 50 mg capsule PO twice daily. The client refuses to take the medication because of nausea. Which of the following actions should the nurse take? Administer a docusate sodium capsule rectally. Withhold the medication. Administer 100 mg docusate sodium with the next scheduled administration. Encourage the client to take the medication as the provider prescribed.

Withhold the medication.

Metformin (Glucophage)

drug class: Antidiabetic Agent, Biguanide MOA: decreases hepatic glucose production, decreases intestinal absorption of glucose, and improves insulin sensitivity by increasing peripheral glucose uptake and utilization. Uses: Diabetes mellitus, type 2 (hyperglycemia cannot be managed with diet and exercise)

Alendronate (Fosamax)

drug class: Bisphosphonate MOA: :inhibits bone resorption via actions on osteoclasts or on osteoclast precursors → indirect increase of bone density uses: paget disease, osteoporosis

Medroxyprogesterone acetate (Depo-Provera)

drug class: Progestin Contraceptive MOA: transforms a proliferative endometrium into a secretory endometrium; inhibits secretion of pituitary gonadotropins, which prevents follicular maturation and ovulation and causes endometrial thinning Uses: contraception, endometriosis pain

Tamsulosin (Flomax)

drug class: alpha 1 blocker MOA: antagonist of alpha1A-adrenoreceptors in the prostate; blocking them leads to relaxation of smooth muscle in the bladder neck and prostate causing an improvement of urine flow and decreased symptoms of BPH. uses: BPH

Testosterone transdermal (AndroGel)

drug class: androgens MOA: a gel that uses testosterone is absorbed via the skin to help low testosterone levels uses: Hypogonadotropic Hypogonadism (HH; congenital or acquired); primary hypogonadism

Ondansetron (Zofran)

drug class: antiemetic MOA: Ondansetron is a selective 5-HT3-receptor antagonist which blocks serotonin, a natural substance that may cause nausea and vomiting. indications: Cancer chemotherapy-induced nausea and vomiting

Prednisone

drug class: corticosteroid MOA: decreased vasodilation and permeability of capillaries, as well as decreased leukocyte migration to sites of inflammation indications: to treat the symptoms of low corticosteroid levels

Docusate sodium (Colace)

drug class: laxative/stool softener MOA: It works by increasing the amount of water the stool absorbs in the gut, making the stool softer and easier to pass. uses: constipation

Omeprazole (Prilosec)

drug class: proton pump inhibitor MOA: It inhibits the parietal cell H+ / K+ ATP pump, the final step of acid production. Uses: is used to treat certain stomach and esophagus problems (such as acid reflux, ulcers).

Levothyroxine (Synthroid)

drug class: thyroid hormone MOA: It replaces or provides more thyroid hormone, which is normally produced by the thyroid gland. Uses: hypothyroidism/Pituitary thyrotropin-stimulating hormone suppression

A nurse is preparing to administer amikacin IM to a client who weighs 176 lb. the medication reference states that the safe dosage range is 7.5 to 15 mg/kg/day divided every 12 to 24 hr. that is the maximum safe dose the nurse should expect to administer per day?

1200

A nurse is preparing to administer potassium chloride (Kaon-CL) 5 mEq/kg/day PO divided in equal doses every 6 hr to a child who weighs 64 lb. Available is potassium chloride liquid 40 mEq/15 mL. How many mL should the nurse administer per dose?

13.6 mL

A nurse is preparing to administer a loading dose of phenytoin (Dilantin) 5 mg/kg/day divided equally ever 12 hr for an infant who weighs 12 lb 4 oz. How many mg should the nurse administer per dose?

13.9 mg

A nurse is preparing to administer ethosuximide 750 mg PO daily. Available is ethosuximide syrup 250 mg/tsp. How many mL should the nurse administer?

15 mL

A nurse is preparing to administer furosemide 40 mg PO in 0.5 oz of orange juice. How many mL of orange juice should the nurse administer?

15 mL

A nurse is preparing to administer lamivudine 150 mg PO every 12 hr. Available is lamivudine oral solution 10 mg/mL. How many mL should the nurse administer per dose?

15 mL

A nurse is preparing to administer rifampin 0.6 g PO daily. Available is rifampin 150 mg capsules. How many capsules should the nurse administer?

4 capsules

A nurse is preparing to administer 0.9% sodium chloride 2,500 mL to infuse over 12 hr. The nurse should administer how many L?

2.5 L

a nurse is preparing to administer levothyroxine 0.25 mg iv bolus stat. available is levothyroxine injection 100 mcg/ml. how many ml should the nurse administer?

2.5 mL

A nurse is preparing to administer dextrose 5% in water 2 L to infuse over 6 hr. The nurse should administer how many mL?

2000 mL

A nurse is preparing to administer potassium gluconate 2 mEq/kg PO every 12 hr to a child who weighs 50 lb. How many mEq should the nurse administer per dose?

22.7

a nurse is preparing to administer cefotaxime 1 g iv bolus every 12 hr. available is cefotaxime injection 1 g/50 ml. how many mL should the nurse administer per dose?

50 mL

A nurse is preparing to administer haloperidol 5 mg IM at 1715. At which of the following times should the nurse administer the medication? 7:15 a.m. 5:15 a.m. 7:15 p.m. 5:15 p.m.

5:15 p.m.

A nurse is preparing to administer ampicillin 50 mg/kg/day PO divided in equal doses every 8 hr to a child who weighs 39.6 lb. available is 250 mg/5mL how many mL should the nurse administer per dose?

6 mL

A nurse is preparing a medication and calculates the dosage as 6.25 mL. Rounding this amount to the nearest tenth, the nurse should administer how many mL?

6.3 mL

The pregnant client tells the nurse that her prescribed medication is not as effective as it was before her pregnancy. What is the best response by the​ nurse? A. ​"This is because your blood volume has​ increased." B. ​"Maybe the medication has​ expired; check the​ label." C. ​"This is because your baby is receiving part of the​ medication." D. ​"Tell me how you have been taking your​ medication."

A. ​"This is because your blood volume has​ increased."

healthcare provider has prescribed a chemotherapeutic drug for a client with cancer. This drug commonly causes loss of hair. The client asks the​ nurse, "Will all of my hair fall​ out?" What is the most therapeutic response by the​ nurse? A. ​"Yes, that is one of the expected side effects of this​ medication." B. ​"It might. Have you discussed this with your healthcare​ provider?" C. ​"Don't worry, we can recommend an excellent wig company if need​ be." D. ​"We are not really​ sure; applying an ice bag to your head may​ help."

A. ​"Yes, that is one of the expected side effects of this​ medication."

A nurse is reviewing a client's prescriptions. The nurse should contact the provider to clarify which of the following prescriptions? Ampicillin 100 mg/kg/day by mouth in 4 equally divided doses Phenytoin 300 mg by mouth every 12 hours Metronidazole 500 mg IV bolus every 6 hr Acetaminophen 325 mg every 6 hr PRN for headache

Acetaminophen 325 mg every 6 hr PRN for headache

A nurse discovers a medication error in which the client received twice the prescribed amount of medication. Which of the following actions should the nurse take first? Notify the provider. Complete an incident report. Assess the client. Report the error to the nurse manager.

Assess the client

A nurse manager is reviewing a client's medical record and discovers that the client received a double dose of a prescribed medication. Which of the following actions should the nurse manager take first? Complete an incident report. Notify the provider about the medication error. Assess the client for adverse effects. Report the error to the risk manager.

Assess the client for adverse effects.

A series of category E medication errors have occurred on a hospital unit. When discussing this trend with the nursing​ staff, the risk manager would list which characteristics of a category E​ error? Select all that apply. A. The error contributed to the death of a client. B. A client was harmed. C. No interventions to sustain life were required as a result of the error. D. Harm to the client was permanent. E. The​ client's hospitalization was prolonged as a result of the error.

B. A client was harmed. C. No interventions to sustain life were required as a result of the error.

The African American client has panic​ attacks, is​ suicidal, and is on an inclient psychiatric unit. The healthcare professional prescribes sertraline​ (Zoloft) and clonazepam​ (Klonopin). The client refuses the drugs. The client also requests to have herbs and African objects in his room to​ "remove the​ curse." What is the priority action by the​ nurse? A. Allow the request after the client signs a release of responsibility to avoid litigation. B. Allow the request without seeking further information from the client. C. Allow the request as long as the herbs and objects do not pose a safety risk for the client or other clients. D. Allow the request after all members of the treatment team agree to it.

C. Allow the request as long as the herbs and objects do not pose a safety risk for the client or other clients.

A student nurse asks a nurse how specialty supplements differ from herbal products. What is the​ nurse's best​ response? A. Specialty supplements are generally targeted for more general conditions. B. Specialty supplements must be prescribed. C. Herbal products are more specific in their actions. D. Specialty supplements can come from animal sources.

D. Specialty supplements can come from animal sources.

A nurse is preparing to administer an oral medication. Which of the following actions should the nurse take? Select all that apply. Provide client education about the medication. Check the expiration date of the medication. Verify the dosage of the medication. Call the client by name to confirm their identity. Ask the client if they have any allergies.

Provide client education about the medication. Check the expiration date of the medication. Verify the dosage of the medication. Ask the client if they have any allergies.

A nurse is providing teaching regarding medication administration to a group of newly licensed nurses. Which of the following is a legal responsibility of a nurse? Prescribing the correct dosage Modifying the medication regimen Reporting medication errors Delegating administration to assistive personnel

Reporting medication errors

A nurse is caring for a client who is to receive topiramate XR 100 mg PO daily. The client tells the nurse that the capsule is too hard to swallow. Which of the following actions should the nurse take? Crush the contents of the capsule to administer in a small amount of pudding. Request extended-release sprinkles from the pharmacy. Ask the charge nurse to clarify the prescription with the provider. Withhold the medication until the time for the next dose.

Request extended-release sprinkles from the pharmacy.

A nurse is transcribing a provider's prescription for a client. The prescription reads morphine 2 mg IV bolus at 1400. The nurse should recognize this as which of the following types of medication orders? Routine order Stat order PRN order Single order

Single order

A nurse is teaching a newly licensed nurse about crushing medications. The nurse should explain that which of the following medications can be crushed? Extended-release oxycodone Sublingual nitroglycerine Enteric-coated aspirin Sucralfate tablets

Sucralfate tablets

Diphenhydramine (Benadryl)

drug class: antihistamine MOA: inverse agonist at the H1 receptor, thereby reversing effects of histamine on capillaries, reducing allergic reaction symptoms. indications: used to relieve symptoms of allergy, hay fever, and the common cold.

a nurse is preparing to administer vasopressin 5 units im to a client. available is vasopressin 20 units/mL. How many mL should the nurse administer?

0.25 mL

a nurse is preparing to administer morphine 4 mg iv bolus every 3 hr prn pain. Available is morphine injection 10 mg/ml. How many mL should the nurse administer per dose?

0.4 mL

A nurse is preparing a medication and calculates the dosage as 0.547 mL. Rounding this amount to the nearest hundredth, the nurse should administer how many mL?

0.55 mL

A nurse is preparing to administer a time - crucial medication to a client at 0800. Which of the following times are appropriate for the nurse to administer the medication? Select all that apply. 0700 0745 0830 0845 0900

0745 0830

A nurse is preparing to administer digoxin 0.125 mg PO at 9:00 a.m. At which of the following times should the nurse administer the medication? 1500 0900 1700 2100

0900

A nurse is preparing to administer digoxin 0.25 mg iv bolus. Available is digoxin injection 0.25mg/ml. How many ml should the nurse administer?

1 mL

A nurse is preparing to administer sucralfate 1 g PO twice daily to a client. Available is sucralfate 1,000 mg tablets. How many tablets should the nurse administer per dose?

1 tablet

A nurse is preparing to administer gluconate 3 mEq/kg/day PO divided in equal doses every 6 hr to a preschooler who weighs 42 lb. Available is potassium gluconate liquid 20 mEq/15 ml. How many ml should the nurse administer?

11 mL

A nurse is preparing to administer potassium chloride 15 mEq PO every 12 hr. Available is potassium chloride liquid 20 mEq/15 mL. How many mL should the nurse administer per dose?

11 mL

The client takes several prescription medications and asks the nurse about using complementary and alternative medicine​ (CAM). What is the best reply by the​ nurse? A. ​"CAM is a good idea that you should discuss with your healthcare​ provider." B. ​"CAM has not been demonstrated to help with your kind of​ symptoms." C. ​"CAM is an approach that might reduce your need for​ medications." D. ​"CAM might​ help, but you will still need your​ medications."

C. ​"CAM is an approach that might reduce your need for​ medications."

A nurse is preparing to administer amlodipine 10 mg PO once per day to a client. The amount available is amlodipine 2.5 mg/tablet. How many tablets should the nurse administer?

4 tablets

A nurse is preparing to administer 650 mg of acetaminophen syrup to a client. The amount available is acetaminophen syrup 160mg/5mL. How many tsp should the nurse administer per dose?

4 tsp

A nurse is caring for a client following a bioterrorism attack. The client has experienced initial nausea and vomiting followed by weight loss and eventual thyroid cancer. What was the most likely causative​ agent? A. Ionizing radiation B. Chemical agent C. Viral agent D. Bacterial agent

A. Ionizing radiation

A pregnant client suspected of drug abuse is admitted to the emergency department. The nurse plans to teach the client about which complications associated with drug use during​ pregnancy? Select all that apply. A. Preterm birth B. Low birth weight C. Birth defects D. Allergies to narcotics E. Increased labor

A. Preterm birth B. Low birth weight C. Birth defects

The nurse is beginning medication reconciliation for a newly admitted client. What should the nurse include in this​ list? Select all that apply. A. The client takes ibuprofen for an occasional headache. B. The client mixes a powdered form of vitamin C into his morning orange juice. C. The client applies essential oils to his forehead to help with his or her allergies. D. The client drinks milk fortified with vitamin D. E. The client takes a prescription medication for osteoporosis once a week.

A. The client takes ibuprofen for an occasional headache. B. The client mixes a powdered form of vitamin C into his morning orange juice. C. The client applies essential oils to his forehead to help with his or her allergies. E. The client takes a prescription medication for osteoporosis once a week.

A client experiencing occasional minor digestive problems asks the nurse about herbal therapy. What is the best response by the​ nurse? Select all that apply. A. Wheat grass B. Green tea C. Ginger D. Grape seed E. Stevia

A. Wheat grass B. Green tea C. Ginger

The nurse recognizes that agency system checks are in place to decrease medication errors. Who commonly collaborates with the nurse on checking the accuracy of the medication prior to​ administration? A. The nursing unit manager B. The pharmacist C. The nursing supervisor D. The healthcare provider

B. The pharmacist

The nurse is providing education about warfarin​ (Coumadin) to a patient of Asian ancestry. The nurse determines that learning has occurred when the client makes which​ statement? A. ​"I may need more medication than someone from a different ethnic​ group." B. ​"I may need less medication than someone from a different ethnic​ group." C. ​"I may need to have more frequent blood​ tests." D. ​"I may need to have less frequent blood​ tests."

B. ​"I may need less medication than someone from a different ethnic​ group."

A nurse is caring for a client who has been exposed to a nerve agent. Which antidote will the nurse plan to​ administer? A. Apomorphine B. Acetate of ammonia C. Atropine D. Hydroxyzine​ (Vistaril)

C. Atropine

A nurse is caring for a client admitted to the emergency department following a drug overdose. The nurse anticipates which of the following prescriptions to enhance removal of the poison from the​ client? Select all that apply. A. Administering activated charcoal. B. Changing the pH of urine. C. Preparing the client for enemas. D. Inserting a nasogastric tube. E. Preparing corticosteroids for administration.

A. Administering activated charcoal. B. Changing the pH of urine. C. Preparing the client for enemas. D. Inserting a nasogastric tube.

The toddler refuses to take his oral medication. What is the best suggestion to the mother from the nurse for ensuring the toddler receives his​ medication? A. ​"Tell him you will buy him a toy if he takes the​ medication." B. ​"Crush the tablet and mix it with a small amount of​ jam." C. ​"Crush the tablet and mix it with​ milk." D. ​"Tell him he will be punished if he does not take the​ medicine."

B. ​"Crush the tablet and mix it with a small amount of​ jam."

A nurse is preparing to administer glipizide 5 mg PO AC. At which of the following times should the nurse administer the medication? After meals Before meals As needed At bedtime

before meals

A nurse is preparing to administer famotidine 20 mg PO every 12 hr. Available is famotidine 40 mg tablets. How many tablets should the nurse administer per dose?

0.5 tablets

When teaching the client about a new​ medication, the nurse should include which​ information? Select all that apply. A. Adverse effects that can be expected B. Which adverse effect to report to the healthcare provider C. The​ drug's therapeutic action D. Chemical composition of the drug E. Name of the drug manufacturer

A. Adverse effects that can be expected B. Which adverse effect to report to the healthcare provider C. The​ drug's therapeutic action

The patient is from an Arab culture and is in labor and delivery. Her husband insists he must stay with her and will not allow her to receive any analgesia during the experience. What is the best action by the​ nurse? A. Allow the request but inform the husband that the healthcare provider will make the final decision about analgesia B. Allow this request and be available in the event the request changes. C. Inform the husband that it is his​ wife's choice whether or not to receive analgesia. D. Inform the husband that he must sign a release of responsibility to avoid future litigation against the hospital

B. Allow this request and be available in the event the request changes.

A client asks a nurse what the common complications are due to​ drug-herbal interactions. How would the nurse​ respond? A. Hair growth B. Blood coagulation C. Urine output D. Vision loss

B. Blood coagulation

A nurse is teaching a group of student nurses. Which of the following best indicates an ethnic characteristic that can affect​ pharmacotherapy? A. Diet B. Genetic differences C. Alternative therapies D. Health beliefs

B. Genetic differences

A nurse is caring for a client being treated for seizures. The nurse would be most concerned if the client is also taking which herbal​ product? A. Ginger B. Ginkgo C. Goldenseal D. Feverfew

B. Ginkgo


Related study sets

English Reading Quiz "Silent Spring

View Set

nur 116 - Davis Advantage / Edge - Seizures

View Set

Behavioral Observation and Screening (BOSR)

View Set

Tissue Integrity Sherpath Questions

View Set

Arjun's khan academy programming unit test

View Set